BRS – Pediatrics: Infectious Diseases

BRS – Pediatrics: Infectious Diseases

Source: BSR Pediatrics, 2019

I. General Approach to the Child With Possible Infection

A. Detailed history and physical examination are essential.
1. History should include details about the present illness and significant past medical history. Important components of the history include chronic, recurrent, or life- threatening infections; travel; animal contact; insect (tick, mosquito, or flea) bites; medications; recent and past immunizations; unusual food ingestions (including raw meat and unpasteurized dairy products); and risk of infection with human immunodeficiency virus (HIV; e.g., intravenous drug use, remote history of blood transfusion, unprotected intercourse).
2. Physical examination should be comprehensive, with special attention to general appearance and mental status rashes, skin manifestations of endocarditis (see Chapter 8, section V.C), hepatosplenomegaly, evidence of joint effusion, and lymph node enlargement.
B. Use of the laboratory. Pathogens may be identified by direct or indirect laboratory methods.
1. Direct methods
a. Cultures for bacteria and viruses
b. Microbiologic stains, including Gram stain, Ziehl–Neelsen stain (acid-fast bacilli),
silver stain (fungal elements), and Wright stain (stool white blood cells [WBCs])
c. Fluorescent antibody–antigen staining for herpes simplex virus (HSV) 1 and 2, varicella-zoster virus (VZV), and respiratory viruses, such as respiratory syncytial virus (RSV), adenovirus, influenza A and B, parainfluenza, and Pneumocystis jiroveci
d. Direct observation, including wet mount for fungal elements and Trichomonas vaginalis and dark-field microscopy for Treponema pallidum
e. Polymerase chain reaction (PCR) is available for identification and quantification of many pathogens and has replaced fluorescent antibody–antigen staining in most centers. It can identify many respiratory pathogens, including Mycoplasma pneumoniae, Bordetella pertussis and Bordetella parapertussis, RSV, adenovirus, all of the herpes viruses (HSV, cytomegalovirus [CMV], Epstein–Barr virus [EBV], VZV, human herpes virus [HHV] 6), parvovirus, HIV, and rickettsial diseases, among others.
2. Indirect methods
a. Intradermal skin testing for Mycobacterium tuberculosis (TB) and Coccidioides immitis (Note: Skin testing for C. immitis, the agent of valley fever, is not widely used, and diagnosis is now more commonly made by sensitive and specific serologic testing.)
b. Antibody testing for viruses (including EBV, CMV, VZV, and HIV); Toxoplasma gondii; Bartonella henselae; rickettsial diseases, including Rocky Mountain spotted fever and murine typhus; Borrelia burgdorferi (the agent of Lyme disease); and Mycoplasma pneumoniae
3. Nonspecific laboratory indications of infection typically include elevation of acute-phase reactants, such as C-reactive protein (CRP) and erythrocyte sedimentation rate (ESR), as well as elevation of the peripheral WBC count.

II. Evaluation of the Child With Fever
A. Fever in children is defined as a rectal temperature of 38°C (100.4°F) or higher. Temperatures taken by axillary, oral, temporal, or tympanic methods may be less accurate.
B. Evaluation for serious bacterial infection (SBI) (e.g., meningitis, pneumonia, sepsis, bone and joint infections, urinary tract infection, and enteritis) should occur in the following high-risk groups:
1. Young infants, especially those younger than 28 days, because of immaturity of their immune system
2. Older infants with high fevers (temperature > 39°C [102.2°F]) who are ill-appearing
3. Infants and children who are immunodeficient. This includes those who have sickle cell disease (or are asplenic for another reason) or have underlying chronic liver, renal, pulmonary, or cardiac disease. This also includes children who are receiving immunosuppressive therapy, such as chemotherapy, or immunomodulating agents for autoimmune conditions, such as inflammatory bowel disease or juvenile idiopathic arthritis (JIA).
C. Evaluation of fever in infants < 3 months
1. Epidemiology. 1–7% of infants with temperature >39°C have a SBI. The highest risk is seen in the first 28 days of life, although the incidence of early onset neonatal sepsis (sepsis in the first week of life) has been significantly reduced as a result of maternal screening and empiric antibiotic treatment during labor and delivery for women colonized by group B streptococcus. Introduction of vaccines for Haemophilus influenzae type b (HIB) and Streptococcus pneumoniae have further decreased the incidence of SBI in infants <3 months of age.
2. Etiology
a. Infections may be acquired in utero (i.e., transplacentally), during passage through the birth canal, or postnatally in the nursery or at home.
b. Viruses are the most common organisms causing infection.
c. Common bacterial organisms causing infection in this age group are summarized in Table 7-1.
3. Clinical features of infection in young infants are often nonspecific and include fever, diminished appetite, irritability, excessive sleepiness, cough, rhinorrhea, vomiting, diarrhea, and apnea.
4. Diagnosis. Clinical and laboratory features may be used to identify infants at low risk for SBI.
a. Infants who are well-appearing and previously healthy, who have had no recent antibiotic therapy, and who have no site of focal infection on examination are at low risk for SBI. Infants who are ill-appearing or toxic, and who have evidence of a focal infection on examination, are at higher risk for SBI.
b. Laboratory evaluation should include complete blood count (CBC), blood culture, urinalysis, urine culture, chest radiograph if tachypnea or respiratory distress is present or if the peripheral WBC count is ≥20,000 cells/mm3, and analysis of the cerebrospinal fluid (CSF). The following are the criteria that indicate an infant is at low risk for SBI:
1. WBC >5000 and <15,000 cells/mm3
2. Absolute band count <1500 cells/mm3
3. Normal urinalysis (<10 WBCs per high-power field)
4. If diarrhea is present, <5 WBCs per high-power field on stool Wright stain
5. Normal CSF

5. Management
a. Hospitalization is required for:
1. All infants ≤ 28 days of age
2. Infants between 29 days and 3 months of age with any of the following:
a. Toxic appearance on examination
b. Suspected meningitis
c. Pneumonia, pyelonephritis, or bone and soft tissue infections, which are unresponsive to oral antibiotics
d. Patients in social circumstances in which there is uncertain outpatient care and follow-up
b. Antibiotic management is based on age, risk factors for infection, and etiology.
1. Infants ≤28 days of age require intravenous antibiotics in the hospital until cultures of blood, urine, CSF, and stool (if diarrhea is present) are negative, typically after 48 hours.
2. Infants 29 days–3 months of age who satisfy low-risk clinical and laboratory criteria [see section II.C.4] appear clinically well and who have transportation and good outpatient follow-up may be managed safely with or without outpatient parenteral antibiotic therapy (i.e., intramuscular ceftriaxone), and may not require a lumbar puncture if they are well appearing. They may also be observed in the hospital without antibiotic administration and discharged when cultures are negative. If antibiotics are to be administered, a full evaluation (blood, urine, and CSF cultures) should be obtained before the initiation of antibiotics.
3. Infants 29 days–3 months of age who do not meet low-risk clinical and laboratory criteria, but appear clinically well, can also be managed as outpatients but should receive parenteral empiric antibiotic therapy (i.e., intramuscular ceftriaxone) following collection of blood, urine, and CSF cultures.
4. Infants who appear ill, irrespective of laboratory data, should be hospitalized for parenteral therapy while cultures are pending.
5. Recommended parenteral antibiotic therapy for hospitalized infants is presented in Table 7-1.
D. Evaluation of fever in children aged 3–36 months
1. Epidemiology. Before the widespread immunization with vaccines for HIB and pneumococcal conjugate vaccine (PCV13), the risk of SBI was 3–10%. The likelihood of bacteremia was as high as 5–10% in well-appearing children with temperature >39°C and a peripheral WBC count >20,000 cells/mm3. With the introduction of these vaccinations, the risk has decreased to less than 1% in a well-appearing child with fever. This reduction has changed the recommendations for management of immunized infants without a source on examination. Infants who have not received the primary booster series of immunizations, including PCV13 and HIB, should be considered incompletely immunized and at higher risk for SBI.
2. Etiology (see Table 7-1). S. pneumoniae is the most common organism. HIB is less common as a result of the introduction of HIB vaccine. Other pathogens include Escherichia coli, particularly in the setting of a probable urinary tract infection, and Staphylococcus aureus (including methicillin-resistant S. aureus [MRSA]) in the setting of skin and soft tissue infection.
3. Diagnostic evaluation and management are based on the degree of the fever and whether the child appears toxic.
a. If the child is toxic or ill-appearing, a complete evaluation for sepsis, intravenous

antibiotics, and hospitalization are required.
b. If the child is nontoxic-appearing and the temperature is <39°C (<102.2°F), and immunizations are up to date, no laboratory tests are required and the child may be observed closely at home.
c. If the child is nontoxic-appearing and has persistent fever (>3 days) with a temperature > 39°C (>102.2°F), the following studies are suggested:
1. Urine culture for male children <12 months of age and female children
<2 years of age
2. CBC with differential, and blood culture if WBC count is >15,000 cells/mm3
3. Chest radiograph if respiratory distress, rales, or tachypnea is present, or if the peripheral WBC count is >20,000 cells/mm3
4. Stool culture if there is blood or mucus in the stool, or if there are ≥5 WBCs per high-power field on Wright stain
5. Empiric antibiotics for all children or only for those whose WBC count is
>15,000 cells/mm3, with re-evaluation in 24–48 hours
d. Recommended parenteral antibiotic therapy for hospitalized infants and children is presented in Table 7-1.

Table 7-1
Typical Bacterial Pathogens and Empiric Antibiotics for Infants and Children with Suspected Sepsis or Meningitis

Age Bacterial Pathogens Empiric Intravenous Antibiotics
0–1 month Group B streptococcus Ampicillin + gentamicin or ampicillin + cefotaxime
Escherichia coli
Listeria monocytogenes
1–3 months Group B streptococcus Ampicillin + cefotaxime (add vancomycin* if bacterial meningitis is suspected)
Streptococcus pneumoniae
Listeria monocytogenes
3 months–

3 years

Streptococcus pneumoniae Cefotaxime (add vancomycin*if bacterial meningitis is suspected)
Neisseria meningitidis Haemophilus

influenzae type b

3 years–adult Streptococcus pneumoniae Cefotaxime (add vancomycin*if bacterial meningitis is suspected)
Neisseria meningitidis

*Vancomycin is added to empiric coverage if bacterial meningitis is suspected to cover for highly resistant S. pneumoniae.

III. Fever of Unknown Origin (FUO)
A. Definition. Fever of unknown origin (FUO) is a term used to describe a fever lasting longer than 8 days to 3 weeks (experts disagree about the length of time of fever necessary to diagnose fever as FUO). With current diagnostic advances, the most accepted definition of FUO is a fever for at least 8 days in a child when prior history, physical examination, and preliminary laboratory evaluation have all failed to lead to a diagnosis.
B. Etiology
1. Most children with FUO do not have a rare illness but rather a common infection with an unusual presentation.
2. The differential diagnosis is extensive (Table 7-2).
3. One-fourth of cases of FUO resolve spontaneously without a diagnosis having been made.
4. Noninfectious causes such as connective tissue disease, autoimmune disease, and malignancy should also be considered.
C. Evaluation should include the following:
1. Comprehensive history, focusing on a thorough review of systems (especially weight loss; rashes; stool patterns; and the height, duration, and pattern of fever), past medical and surgical history (including prior blood transfusions), travel history, animal exposure, and family and social history.
2. Detailed physical examination, focusing on general appearance and growth curves, skin and mucous membrane findings, presence of lymphadenopathy and hepatosplenomegaly, and evaluation of joints and bones.
3. Laboratory studies are based on the history and physical examination, and the initial evaluation often includes the following:
a. CBC with differential to evaluate for infection or leukemia
b. ESR or CRP, which are nonspecific indicators of tissue inflammation
c. Serum transaminases to evaluate for hepatitis
d. Urinalysis and urine culture to evaluate for infection
e. Blood cultures to evaluate for bacteremia, including endocarditis
f. Antistreptolysin O (ASO) titer to evaluate for recent streptococcal infection, as seen in rheumatic fever
g. Antinuclear antibody (ANA) and rheumatoid factor (RF) to screen for rheumatic diseases
h. Stool for fecal leukocytes, culture, ova and parasites, and PCR for Clostridium difficile toxin if diarrhea is present
i. Tuberculosis skin test or IGRA (interferon gamma release assay) testing
j. HIV testing (enzyme-linked immunoassay [EIA] and/or PCR)
4. Imaging options may include chest radiography, echocardiography to evaluate for bacterial endocarditis, bone scanning to evaluate for osteomyelitis, gallium scanning to assess for sites of inflammation, and computed tomography (CT) and magnetic resonance imaging (MRI) of specific areas of concern. Abdominal, pelvic, and chest CT scans are frequently performed as part of an extended workup for FUO to localize occult abscesses, to identify lesions in the liver or spleen, and to detect pulmonary infiltrates which are not visualized on a plain chest x-ray.
5. Management. Hospitalization is generally recommended for children with fever for more than 2 weeks to facilitate evaluation and to document fever and coexisting symptoms. Hospitalization can also be helpful to document the fever pattern and associated vital sign changes with fever, and to coordinate the workup. Specific

management is based on the identified cause of the FUO.

Table 7-2
Differential Diagnosis of Fever of Unknown Origin

Infectious disorders (most common cause of FUO)

Occult infection (e.g., pyelonephritis, sinusitis, mastoiditis, or otitis media)

Viral syndromes (e.g., Epstein–Barr, cytomegalovirus, enterovirus, hepatitis B, HIV, or parvovirus B19)

Occult bacteremia (e.g., salmonellosis, tularemia, brucellosis, or gonococcemia)

Bacterial endocarditis

Tuberculosis

Occult abscess (e.g., liver, intra-abdominal, or perinephric)

Musculoskeletal infections (e.g., diskitis, osteomyelitis, or septic arthritis)

Spirochete infections (e.g., Lyme disease, or leptospirosis)

Parasitic infections (e.g., malaria, or toxoplasmosis)

Cat scratch disease (Bartonella henselae)
Rickettsial disease (murine typhus, or Rocky Mountain spotted fever)
Rheumatologic disorders (second most common cause of FUO)

Juvenile idiopathic arthritis (JIA) (including systemic JIA, which is termed Still’s disease)

Kawasaki disease

Systemic lupus erythematosus

Acute rheumatic fever

Polyarteritis nodosum
Malignancy (third most common cause of FUO)

Lymphoma

Leukemia
Periodic fever disorders characterized by spiking fevers at regular monthly intervals. Although these are commonly considered
as FUO, they are more typically of shorter duration (3–5 days) and recur at regular intervals.

Familial Mediterranean fever: fever, peritonitis, pleuritis, and monoarthritis

Periodic fever syndrome or periodic fever, aphthous stomatitis, pharyngitis, cervical adenitis syndrome (PFAPA): typically presenting in early childhood with regularly occurring monthly fevers and associated symptoms. This syndrome is self-limited and usually resolves by 5 or 6 years of age.

Cyclic neutropenia: neutropenia at time of fever occurring at regular 21-day intervals

Miscellaneous causes

Inflammatory bowel disease

Sarcoidosis

Drug fever

Factitious disorder
FUO = fever of unknown origin; HIV = human immunodeficiency virus.

IV. Meningitis
A. Definition. Meningitis is the inflammation of the meninges and is classified as bacterial or
aseptic (nonbacterial).
B. Bacterial meningitis
1. Epidemiology
a. Highest incidence of bacterial meningitis is during the first month of life.
b. Overall incidence during childhood has declined dramatically owing to the introduction of conjugate vaccines for S. pneumoniae and HIB. Before vaccination, HIB was the leading cause of bacterial meningitis outside the newborn period.
c. Risk factors for bacterial meningitis include the following:
1. Young age
2. Immunodeficiency (e.g., asplenia, humoral-mediated immunodeficiency, and terminal complement deficiency, which predisposes individuals to infections with Neisseria meningitidis)
3. Anatomic defects (e.g., basilar skull fracture, ventriculoperitoneal shunt, cochlear implants), which increase the risk of meningitis caused by S. pneumoniae owing to direct extension from the nasopharynx and ear.
2. Etiology. Causes of infection are based on the age of the child (see Table 7-1).
3. Clinical features
a. Infants and young children often have minimal and nonspecific signs and symptoms (e.g., poor feeding, irritability, lethargy, respiratory distress). Fever may be absent or minimal. A bulging fontanelle may be found on physical examination.
b. Older children often present with fever and signs suggestive of meningeal irritation.
1. Alteration in level of consciousness, with irritability, somnolence, or obtundation
2. Nuchal rigidity and positive Kernig (inability to extend the knee when the thigh is flexed 90° at the hip) and Brudzinski (the thighs and legs involuntarily flex when the neck is flexed) signs. These signs indicate inflammation of sensory nerves and are less reliably present in infants and young children.
3. Seizures (20–30% of children have a seizure before or at the time of admission)
4. Photophobia
5. Emesis
6. Headache. Both emesis and headache are symptoms of increased intracranial pressure.
4. Diagnosis. Index of suspicion for bacterial meningitis should be especially high in febrile, irritable infants. Evaluation of all children with suspected bacterial meningitis should include the following:
a. Lumbar puncture, which may demonstrate
1. Pleocytosis with a predominance of neutrophils. The CSF WBC count often exceeds 5000 cells/mm3, but early in the course the cell count may be lower.
2. Hypoglycorrhachia (low CSF glucose)—ratio of CSF glucose to serum glucose
<0.40 and CSF glucose typically <40 mg/dL.
3. Increased protein-typically 100–500 mg/dL
4. Increased intracranial pressure (in older children)
5. Positive Gram stain and/or culture
6. Bacterial antigens may be tested but have a low sensitivity and are not

recommended on a routine basis.
7. Pretreatment with antibiotics may sterilize the CSF culture but should not alter the aforementioned CSF cellular and biochemical profile.
8. See Table 7-3 for a comparison of the CSF profile in bacterial, viral, tuberculous, and fungal meningitis.
9. Interpretation of a traumatic lumbar puncture: Lumbar punctures that contain a large number of red blood cells (RBCs) are difficult to interpret, but a correction can be calculated that accounts for the ratio of WBCs and RBCs in the serum from the patient’s CBC. For patients with a normal peripheral WBC count, a rough estimation is that one WBC in the CSF may account for every 500 RBCs in the CSF.
b. Blood culture is positive in most cases of bacterial meningitis.
c. CT scan with contrast to evaluate for brain abscess is often recommended, especially for patients with focal neurologic findings or altered mental status.
5. Management. Early, empiric treatment of bacterial meningitis is critical.
a. Antibiotic therapy varies based on age of child and the most likely pathogens, as follows:
1. Newborns (0–28 days): Ampicillin plus aminoglycoside or a third-generation cephalosporin. Intravenous acyclovir for possible HSV infection should also be considered for ill neonates, especially those presenting with apnea, seizures, or cutaneous vesicles.
2. Young infants (1–3 months): Ampicillin (for Listeria monocytogenes coverage) plus a third-generation cephalosporin. Vancomycin should be added if bacterial meningitis is highly suspected, to cover for highly resistant S. pneumoniae.
3. Older infants and children (>3 months): Third-generation cephalosporin. Vancomycin should be added if bacterial meningitis is highly suspected, to cover for highly resistant S. pneumoniae.
b. Corticosteroids given before or with the first dose of antibiotics have been shown to be effective at reducing the incidence of hearing loss in HIB meningitis. Efficacy in other causes of bacterial meningitis has not been as clearly demonstrated. If corticosteroids are given, they should be administered before or with the first dose of antibiotics for maximum benefit, and should only be administered to children outside the neonatal period. Dexamethasone is given for a 2–4-day regimen
(0.6 mg/kg per day) divided into four daily doses.
c. Supportive care. Children presenting with meningitis typically have been ill for a few days and may be moderately dehydrated. Management should include attention to fluids with rehydration to normovolemic status, followed by administration of maintenance fluids with close attention to vital signs, urine output, specific gravity, and serum sodium to monitor for the development of the syndrome of inappropriate release of antidiuretic hormone (SIADH).
6. Complications. Complication rates are highest following meningitis caused by Gram- negative organisms, followed by S. pneumoniae, HIB, and finally, Neisseria meningitidis. Mortality rates range from 5 to 50% depending on the infecting bacterial species.
a. Hearing loss is the most common complication, occurring in up to 25% of patients.
b. Global brain injury occurs in 5–10% of patients.
c. Other complications include SIADH, seizures, hydrocephalus, brain abscess, cranial nerve palsy, learning disability, and focal neurologic deficits.
C. Aseptic meningitis
1. Definition. Aseptic meningitis is the inflammation of the meninges with a CSF

lymphocytic pleocytosis and, if caused by a virus, normal CSF glucose and normal to minimally elevated CSF protein.
2. Etiology (Table 7-4). Most causes of aseptic meningitis are viral.
3. Clinical features may be similar to those found in bacterial meningitis.
a. Symptoms of viral meningitis may be mild with fever, headache, and emesis, or severe with altered level of consciousness and seizures.
b. Symptoms in aseptic meningitis caused by M. tuberculosis (TB) may be nonspecific initially, with lethargy or irritability. During the second week of illness, signs and symptoms progress rapidly and may include cranial nerve deficits, altered level of consciousness, coma, paraplegia, and eventually death if untreated.
4. Diagnosis (see Table 7-3 for the CSF findings commonly seen in viral, fungal, tuberculous, and bacterial meningitis)
a. Viral meningitis
1. Viral culture of the CSF may be performed; however, the virus may not grow for 10–14 days, if it grows at all.
2. PCR technology is available for the detection of EBV, CMV, HSV, VZV, and enteroviruses.
3. Positive surface cultures for enterovirus from throat and/or rectum may be suggestive in cases of enteroviral meningitis.
b. TB meningitis. CSF findings include lymphocytic pleocytosis, hypoglycorrhachia, and dramatically elevated protein. Brain imaging shows a characteristic basilar enhancement. Positive CSF acid-fast bacilli (AFB) stains (although rarely positive in TB meningitis), positive culture (may take as long as 6 weeks for growth), or positive PCR findings are diagnostic. Fifty percent of patients have a negative chest radiograph and tuberculin skin test at the time of presentation.
c. Diagnostic studies that may identify other causes of aseptic meningitis include CSF rapid plasma reagent (RPR) for syphilis; India ink for fungus (cryptococcus) and cryptococcal antigen tests; and CSF and serum antibody testing for coccidioidomycosis, Lyme disease, rickettsial disease, and cysticercosis.
5. Management
a. Most causes of viral meningitis are self-limited.
b. TB meningitis is treated with four medications, including isoniazid, rifampin, pyrazinamide, and streptomycin or a fluoroquinolone (levofloxacin). Corticosteroids are also commonly used to manage brain inflammation and elevated intracranial pressure.
6. Prognosis of aseptic meningitis is dependent on the causative agent, and ranges from excellent in enteroviral meningitis to poor in TB meningitis (20% mortality in young children).

Table 7-3
Cerebrospinal Fluid Profiles in Meningitis

Pathogen WBC Differential (cells/mm3) Protein Glucose Gram Stain, Culture, and Other

Definitive Tests

Acute bacterial 100–50,000 High Low Positive culture and Gram stain
PMNs predominate
Partially treated bacterial 1000–10,000Monos predominate Normal to high Low normal Negative culture and usually

negative Gram stain

(+) CSF bacterial antigens
Viral 10–1000PMNs early, then monos + lymphsHSV encephalitis may show RBCs Normal to minimally elevated Normal Enterovirus may be recovered by

culture

Enterovirus and HSV may be

identified by PCR

Tuberculosis              10–500Lymphs predominate                        Very high             Low to

very low

AFB smear and culture rarely positive
PCR may be positive
Fungal                        25–500Lymphs predominate                        Normal to            Low

high

Culture may be positive
For cryptococcal meningitis, India ink

and cryptococcal antigen are positive

For coccidioidal meningitis, cocci

antibody is positive

Parameningeal focus (brain

abscess)

10–200 High                     Normal      Negative culture
PMNs or monos predominate

AFB = acid-fast bacilli; PMNs = polymorphonuclear cells; monos = mononuclear cells; lymphs = lymphocytes; CSF = cerebrospinal fluid; PCR = polymerase chain reaction; RBCs = red blood cells; HSV = herpes simplex virus; WBC = white blood cell.

Table 7-4
Causes of Aseptic Meningitis

Viral meningitis (most common cause of aseptic meningitis). If infection also involves the brain, it is termed a
meningoencephalitis.

Enteroviruses (most common cause of viral meningitis in the United States; most common in the summer and fall)

Mumps

Lymphocytic choriomeningitis

Herpes viruses (herpes simplex virus, Epstein–Barr virus, cytomegalovirus, varicella zoster virus, human herpes viruses 6 and 7)
Human immunodeficiency virus Parechoviruses

Viruses that commonly cause encephalitis include arboviruses (St. Louis, Western equine, Eastern equine, and West Nile virus), influenza, and the herpes viruses
Bacterial causes (some bacteria may cause an aseptic picture)

Mycobacterium tuberculosis (most commonly seen in children younger than 5 years of age)

Borrelia burgdorferi (Lyme disease)

Treponema pallidum (syphilis)
Brucella spp.
Fungal causes

Coccidioides immitis

Cryptococcus neoformans

Histoplasmosis capsulatum
Parasitic causes

Taenia solium (etiologic agent of cysticercosis)

Toxoplasma gondii (in immunocompromised patients)
Roundworms: Baylisascaris procyonis causes an eosinophilic meningitis
Naegleria fowleri
Noninfectious causes

Kawasaki disease

Medications (sulfa, intravenous immune globulin [IVIG])

Autoimmune conditions (systemic lupus erythematosus [SLE])

V. Upper Respiratory Infections
A. General concepts. Upper respiratory infections (URIs) account for most pediatric acute illness visits. Although generally benign illnesses, they may cause significant morbidity and parental anxiety.
B. Simple URI (common cold)
1. Etiology. More than 100 viruses have been implicated and include rhinovirus, parainfluenza virus, coronavirus, and RSV.
2. Clinical features
a. Presenting symptoms include low-grade fever, rhinorrhea, cough, and sore throat. Symptoms resolve within 7–10 days.
b. Color of nasal discharge alone does not predict the presence of concurrent sinusitis because purulent nasal discharge may occur early in the course of a URI.
c. Persistent symptoms (>10 days) or persistent fever should prompt the clinician to evaluate for bacterial superinfection (e.g., sinusitis, acute otitis media).
3. Diagnosis is based on clinical features. The viral agent is rarely identified.
4. Management. The most important step is to ensure adequate hydration, particularly in young children, and to exclude more serious disorders, such as sinusitis and acute otitis media. Over-the-counter medications (e.g., antihistamines, mucolytics, cough suppressants, and decongestants) have minimal effectiveness and may cause side effects. Antibiotics have no role in the management.
C. Sinusitis
1. Development of the sinuses. Ethmoid and maxillary sinuses form in the third to fourth month of gestation and are present at birth. The sphenoid sinuses develop between 3 and 5 years of age, and frontal sinuses between 7 and 10 years of age.
2. Categories of sinusitis. Sinusitis is divided into acute, subacute, and chronic forms on the basis of duration of symptoms.
3. Diagnosis is based on clinical features. Note that physical examination (particularly sinus transillumination) is unreliable for diagnosis and that imaging is not useful for the initial diagnosis or management of uncomplicated sinusitis.
4. Management of uncomplicated acute bacterial sinusitis includes empiric antibiotic therapy (for example, with amoxicillin-clavulanate) to cover likely pathogens, including
S. pneumoniae, H. influenzae, and M. catarrhalis.
D. Pharyngitis
1. Etiology
a. Viral causes include those viruses associated with simple URIs, as well as
coxsackievirus, EBV, and CMV.
b. Bacterial causes include Streptococcus pyogenes (group A β-hemolytic streptococcus [GABHS] or “strep throat”), Arcanobacterium hemolyticum, and Corynebacterium diphtheriae (diphtheria).
2. Clinical features. The clinical features of viral pharyngitis and GABHS pharyngitis overlap, resulting in difficulty differentiating between the two conditions solely on the basis of history and physical examination.
a. Viral pharyngitis may present with simple URI symptoms. Tonsillar exudates may also be present. Certain viral infections may have the following specific findings:
1. Children with EBV pharyngitis may present with enlarged posterior cervical lymph nodes, malaise, and hepatosplenomegaly.
2. Children with coxsackievirus pharyngitis may present with painful vesicles or ulcers on the posterior pharynx and soft palate (herpangina). Blisters may

also be present on the palms and soles (hand-foot-mouth disease).
b. Bacterial pharyngitis
1. GABHS pharyngitis (strep throat) is usually seen in school aged children (5– 15 years of age) and most commonly in the winter and spring. Although it is difficult to distinguish GABHS pharyngitis from viral pharyngitis on the basis of signs and symptoms, GABHS infection has the following characteristics:
a. Lack of other URI symptoms (e.g., rhinorrhea, cough)
b. Exudates on the tonsils, petechiae on the soft palate, strawberry tongue, and enlarged tender anterior cervical lymph nodes
c. Fever
d. Scarlatiniform rash in some patients [see section IX.A.5] e. Complications of GABHS infection are described in section IX.A.5.g.
2. Diphtheria is extremely rare in developed nations because of universal vaccination. Patients present with low-grade fever and a gray, adherent tonsillar membrane. Toxin-mediated cardiac and neurologic complications may also develop.
3. Diagnosis
a. Patients with suspected GABHS pharyngitis should undergo culture (gold standard) or antigen testing (“rapid strep test”) to confirm GABHS pharyngitis and to avoid the overuse of antibiotics.
b. Because 5% of the population carries GABHS in the pharynx (GABHS carriers), culture or rapid antigen testing should be limited to symptomatic patients who lack concomitant viral symptoms.
4. Management
a. Management of viral pharyngitis is supportive and includes analgesics and the maintenance of adequate hydration.
b. Management of GABHS pharyngitis includes oral penicillin VK or amoxicillin, a single dose of intramuscular benzathine penicillin, or for penicillin-allergic patients, a macrolide.
c. Management of severe EBV pharyngitis, with pending airway obstruction, may sometimes include corticosteroids.
d. Diphtheria is treated with oral erythromycin or parenteral penicillin, and a specific antitoxin that is available from the Centers for Disease Control and Prevention. Respiratory isolation is very important to prevent spread of infection.
E. Acute otitis media
1. Definitions
a. Acute otitis media (AOM) is defined as an acute infection of the middle ear space.
b. Otitis media with effusion (OME) is defined as fluid within the middle ear space without signs or symptoms of infection.
2. Etiology. Bacterial pathogens include S. pneumoniae, nontypeable H. influenzae, and
Moraxella catarrhalis. Viruses causing URIs also commonly cause AOM.
3. Clinical features of AOM
a. AOM usually develops during or after a URI.
b. Symptoms may include fever, ear pain, and decreased hearing. Symptoms are less reliably present in young children.
c. If the tympanic membrane perforates, patients may report pus or fluid draining from the ear.
4. Diagnosis
a. Proper diagnosis of AOM depends on the identification of fluid within the middle ear space in the presence of symptoms of infection.

1. Pneumatic otoscopy to identify abnormal movement of the tympanic membrane, and therefore fluid within the middle ear, is an essential component of the physical examination and is the most reliable method of detecting middle ear fluid.
2. Erythema and loss of tympanic membrane landmarks are unreliable methods of identifying fluid within the middle ear space.
b. Although not routine, identification of the bacterial etiology may be made by tympanocentesis.
c. A perforated tympanic membrane with purulent discharge within the external auditory canal is also consistent with the diagnosis of AOM.
5. Management
a. Antibiotics are often prescribed for AOM; however, they are not always indicated, especially in older children, because most cases of AOM resolve spontaneously without complications.
b. Initial antibiotic therapy, if used, is usually amoxicillin. If the patient attends a day care facility or has received antibiotics within the previous 1–2 months, then the likelihood of infection with penicillin-resistant S. pneumoniae increases. Initial therapy may then include high-dose amoxicillin, amoxicillin–clavulanic acid, or a cephalosporin. Macrolides may be used in penicillin-allergic patients.
c. Antibiotics are not indicated for OME.
F. Otitis externa
1. Definition. Otitis externa (OE) is defined as an infection of the external auditory canal
(EAC).
2. Pathogenesis. Factors that interfere with the EAC-protective mechanisms (e.g., cerumen removal, trauma, maceration of the skin from swimming, or excessive moisture or humidity) predispose to OE.
3. Etiology. Pathogens are most commonly Pseudomonas aeruginosa, S. aureus, or Candida albicans. OE can also develop in a patient with a perforated tympanic membrane secondary to AOM.
4. Clinical features. Pain, itching, and drainage from the ear are usually present. Systemic symptoms are usually absent. A history consistent with AOM is helpful in determining whether there has been a tympanic membrane perforation.
5. Diagnosis. Findings on physical examination are the basis of diagnosis. Erythema and edema of the EAC may be present, sometimes with purulent or whitish material within the canal. There may also be tenderness on palpation or movement of the tragus. Visualization of the tympanic membrane is important to exclude perforation. In refractory cases, cultures of infected material may identify the etiologic agent.
6. Management
a. The key to successful management is to restore the EAC to its natural acidic environment.
b. For mild cases of OE (minimal pain and discharge), acetic acid solution may be sufficient to relieve the discomfort and to restore the natural environment of the EAC.
c. For more severe cases, topical antibiotics (sometimes combined with a topical corticosteroid) are prescribed.
d. Perforated AOM complicated by OE is treated with both oral and topical antibiotics.

VI. Middle and Lower Respiratory Infections
Middle and lower respiratory infections, including pneumonia, bronchiolitis, epiglottitis, croup, and bacterial tracheitis, are described in Chapter 9, section III.

VII. Cervical Lymphadenitis
A. Definition. Cervical lymphadenitis is defined as an enlarged, inflamed, tender lymph node or nodes in the cervical area.
B. Etiology and differential diagnosis
1. Localized bacterial infection
a. S. aureus is the most common bacterial agent.
b. S. pyogenes is also common.
c. Mycobacterial infections, including M. tuberculosis and atypical mycobacterium (Mycobacterium avium complex)
d. B. henselae, which causes cat scratch disease [see section XVII.B] 2. Reactive lymphadenitis occurs in response to infections in the pharynx, teeth, and soft tissues of the head and neck.
3. Viral infections, such as EBV, CMV, and HIV, can also cause reactive lymphadenitis in the cervical area.
4. Kawasaki disease may present with unilateral cervical lymphadenitis. A unilateral enlarged cervical lymph node (>1.5 cm in diameter) is one of the diagnostic criteria of Kawasaki disease. [see chapter 16, section II] 5. T. gondii infection may cause a mononucleosis-like illness with cervical lymphadenopathy.
6. Structural lesions in the neck (e.g., branchial cleft cyst, cystic hygroma) can become secondarily infected and may present similarly to cervical lymphadenitis.
C. Clinical Features of Lymphadenitis
1. The infected node is mobile, tender, warm, and enlarged, and the overlying skin is erythematous. Fluctuance may be present.
2. Nodes may be single, or multiple nodes may be clumped together in a mass.
3. Systemic symptoms (e.g., fever) may be present.
D. Diagnosis is based on clinical features.
1. Tests, such as placement of a tuberculin skin test, a CBC with differential, and antibody titers for B. henselae and T. gondii, may be indicated for infected nodes unresponsive to therapy.
2. Antibody titers for EBV, CMV, and HIV serology may be indicated if lymphadenopathy is diffuse and persistent.
3. Imaging studies may help define the anatomy of the cervical area and identify areas of suppuration or abscess that might require surgical drainage. Imaging is essential if there is concern about airway compromise resulting from a deep infection.
E. Management includes empiric antibiotics directed toward the most common organisms (S. aureus and S. pyogenes). Initial treatment may include a first-generation cephalosporin or an antistaphylococcal penicillin for 7–10 days. Intravenous antibiotics are indicated for the toxic- appearing child with lymph adenitis, or for the child who remains symptomatic despite appropriate oral therapy.

VIII. Parotitis
A. Definition. Parotitis is defined as inflammation of the parotid salivary glands.
B. Etiology
1. Mumps and other viruses (e.g., CMV, EBV, HIV, influenza) usually cause bilateral involvement of the parotid gland. Before universal vaccination, mumps was the most common cause of parotitis.
2. Bacterial parotitis (acute suppurative parotitis) is caused by S. aureus and S. pyogenes, and usually results in unilateral parotid involvement. Bacterial parotitis is uncommon during childhood, although children with decreased salivary flow or stone formation are at increased risk.
C. Clinical features include swelling centered above the angle of the jaw with fever. Physical examination of the oropharynx may reveal pus that can be expressed from Stensen’s duct.
D. Diagnosis is based on clinical features and may be confirmed by CT scan.
1. Culture of the drainage from Stensen’s duct may reveal the microbiologic cause of bacterial parotitis.
2. Viral parotitis may be diagnosed by viral serology. Mumps virus may also be detected in the urine.
E. Management
1. Viral parotitis is treated with supportive care and analgesics.
2. Acute suppurative parotitis is treated with antibiotics directed against S. aureus and S. pyogenes. Rarely, surgical excision and drainage are required.
F. Complications
1. Mumps may also result in meningoencephalitis, orchitis and epididymitis, and pancreatitis.
2. Acute suppurative parotitis may result in formation of an abscess and osteomyelitis of the jaw.

IX. Skin and Soft Tissue Infections
A. Bacterial infections
1. Impetigo
a. Definition. Impetigo is a superficial skin infection involving the upper dermis.
b. Etiology. S. aureus is the most common agent, but GABHS (or S. pyogenes) may also cause infection.
c. Clinical features. Honey-colored crusted or bullous lesions are present, commonly on the face, especially around the nares. Fever is generally absent. Infection is easily transmitted.
d. Diagnosis. Visual inspection is the basis of diagnosis. Cultures are not required.
e. Management. Treatment may include topical mupirocin or oral antibiotics, such as dicloxacillin, a first-generation cephalosporin, or clindamycin.
f. Complications. Bacteremia, poststreptococcal glomerulonephritis (treatment of impetigo does not prevent this complication), and staphylococcal scalded skin syndrome (SSSS) are possible complications.
2. Erysipelas
a. Definition. Erysipelas is a skin infection that involves the dermal lymphatics.
b. Etiology is usually GABHS.
c. Clinical features include tender, erythematous skin with a distinct border. The face and scalp are common locations.
d. Diagnosis is by visual inspection.
e. Management includes systemic therapy with antibiotics targeted against GABHS.
f. Complications include bacteremia, poststreptococcal glomerulonephritis, and necrotizing fasciitis.
3. Cellulitis
a. Definition. Cellulitis is a skin infection that occurs within the dermis.
b. Etiology. Causes include GABHS and S. aureus. Infection is usually caused by a break in the skin barrier, allowing bacteria to gain td class=”id” beyond the protective layer of the epidermis.
c. Clinical features. Cellulitis is characterized by erythema, warmth, and tenderness. The infected skin border is indistinct.
d. Diagnosis is by visual inspection. Blood cultures are seldom positive. In more aggressive forms of cellulitis, biopsy and culture of the leading edge of infection may be useful to identify the pathogenic organism.
e. Management includes oral or intravenous antibiotics directed against the typical causative agents, including first-generation cephalosporins or antistaphylococcal penicillins.
4. Important variants of cellulitis
a. Buccal cellulitis is now an uncommon form of cellulitis that presents as a unilateral bluish discoloration on the cheek of a young unimmunized child. Patients are often febrile and may appear toxic. The causative agent is HIB, and blood cultures are often positive. Management includes intravenous antibiotics directed against H. influenzae, usually a second- or third-generation cephalosporin (e.g., cefuroxime or cefotaxime). Patients with buccal cellulitis caused by HIB have a high rate of concomitant bacteremia and meningitis, and therefore blood cultures and CSF cultures should be obtained.
b. Perianal cellulitis occurs as well-demarcated erythema involving the skin around the anus. Children may also present with constipation. The cause is usually

GABHS. Diagnosis is by visual inspection or a positive rectal swab culture for GABHS. Management includes oral antibiotics (e.g., first-generation cephalosporin, dicloxacillin).
c. Necrotizing fasciitis is a potentially fatal form of deep cellulitis. Patients present with pain and systemic symptoms out of proportion to physical findings. Infection extends beyond the underlying fascia into the muscle. Examination may reveal crepitus and hemorrhagic bullae. The cause is polymicrobial and may involve GABHS and anaerobic bacteria. Intravenous antibiotics and surgical debridement are essential components of therapy.
d. Staphylococcal Scalded Skin Syndrome (SSSS) is caused by strains of S. aureus that produce an exfoliative toxin. Presentation includes fever, tender skin, and bullae. Large sheets of skin slough several days after the illness begins, and the Nikolsky sign is present (extension of bullae when pressure is applied to the skin). Management includes good wound care and intravenous antibiotics directed against S. aureus.
5. Scarlet fever
a. Definition. Scarlet fever is a toxin-mediated bacterial illness that results in a characteristic skin rash.
b. Etiology. The infection is caused by strains of GABHS that produce an erythrogenic toxin.
c. Epidemiology
1. Peak incidence is in the winter and spring.
2. Transmission is by large respiratory droplets or by infected nasal secretions.
d. Clinical features
1. The exanthem may develop during any GABHS infection (e.g., impetigo, cellulitis, pharyngitis).
2. Before or during the exanthem, fever, chills, malaise, and often an exudative pharyngitis [see also section V.D] may occur.
3. The exanthem is characterized by the following:
a. Begins on the trunk and moves peripherally
b. The skin is erythematous with tiny skin-colored papules (scarlatiniform appearance) with the texture of sandpaper (sandpaper rash). The rash blanches with pressure.
c. Petechiae are often localized within skin creases in a linear distribution (“Pastia lines”).
d. Desquamation of dry skin occurs as the infection resolves.
e. Diagnosis. The basis of diagnosis is clinical features and a positive throat culture for S. pyogenes (gold standard), or a positive rapid streptococcal tests that detect the GABHS antigen.
f. Management
1. The goal is to prevent development of rheumatic fever.
2. Appropriate antibiotics include oral penicillin VK or amoxicillin, intramuscular benzathine penicillin, or, for penicillin-allergic patients, erythromycin or macrolides.
g. Complications of GABHS infections
1. Poststreptococcal glomerulonephritis may occur several weeks after streptococcal pharyngitis. Patients present with hypertension and cola-colored urine. Antibiotic therapy does not prevent this complication (see Chapter 11, section V.F.1).
2. Rheumatic fever (see Chapter 16, section VI)

3. Poststreptococcal arthritis is characterized by joint symptoms (without other features of rheumatic fever) that may last for weeks. Antibiotic therapy does not prevent this complication.
6. Toxic shock syndrome (TSS)
a. Definition. TSS is a toxin-mediated illness characterized by fever, shock, desquamating skin rash, and multiorgan dysfunction.
b. Etiology and pathogenesis
1. S. aureus is the most common organism associated with TSS, although an increase in GABHS-associated TSS has been reported. In the early 1980s, the majority of TSS cases caused by S. aureus were in young women using tampons. However, only 50% of TSS cases are now related to tampon use.
2. Organisms produce toxins (exotoxins and TSS toxin) that result in the clinical features of TSS.
c. Clinical features and diagnostic criteria (Table 7-5). Patients with TSS may present with a wide variety of signs and symptoms.
d. Management. Treatment includes supportive measures to reverse shock, antistaphylococcal antibiotics, and removal of the nidus of infection (tampon) if present. Intravenous immune globulin (IVIG) may have some benefit.
7. Infection caused by animal and human bites (see Chapter 20, sections IX.B–D)
B. Fungal infections of the skin (Chapter 19, section V.A)
C. Viral infections of the skin (see Chapter 19, section V.C)
D. Ectoparasitic infections of the skin (see Chapter 19, section V.D)

TABLE 7-5
Diagnostic Criteria for Toxic Shock Syndrome Due to Staphylococcus aureus

Diagnostic criteria
(Probable case = five of six criteria; confirmed case = six of six criteria)

1. Temperature >38.9°C

1. Hypotension (SBP <90 mm Hg or < 5th percentile for age for children)

1. Diffuse macular erythroderma (appears similar to sunburn)

1. Desquamation, particularly of the extremities including the palms, soles, fingers, and toes, occurs 10–14 days after onset of illness

1. Multisystem involvement, including three or more of the following:
a. Gastrointestinal—vomiting, diarrhea, and abdominal pain at onset of illness
b. Myalgias or elevated creatine kinase levels > twice the upper limit of normal
c. Hyperemia of the mucous membranes (vaginal, oropharyngeal, or conjunctival hyperemia)
d. Pyuria in the presence of negative urine cultures, or elevated blood urea nitrogen and creatinine to twice normal limit
e. Thrombocytopenia—platelet count < 100,000 mm3
f. Central nervous system—altered level of consciousness, without focal neurologic signs when fever and hypotension are absent.

1. Negative cultures of blood, cerebrospinal fluid, and pharynx (except for positive blood culture for S. aureus) and negative serologic tests for Rocky Mountain spotted fever (RMSF), leptospirosis, or measles.
Other clinical findings may include oral ulcers, acute respiratory distress syndrome, headache, edema, conjunctivitis, disseminated intravascular coagulation, elevated transaminases, hypocalcemia, and hypoalbuminemia.
SBP = systolic blood pressure.

X. Bone and Joint Infections (see Chapter 17, section III.B)

XI. Diarrhea
A. Etiology. Diarrheal diseases and resulting dehydration are among the most common causes of childhood morbidity and mortality worldwide. Specific infectious causes of diarrhea include the following:
1. Viral causes most commonly include rotavirus and norovirus.
a. Rotavirus
1. Epidemiology. Worldwide, rotavirus is the most common infectious agent causing gastroenteritis. This RNA virus is usually seen in the winter months and is easily spread by the fecal–oral route. Incidence has significantly decreased following implementation of routine rotavirus vaccine for infants in the first 6 months of life.
2. Clinical features. Incubation period is 1–3 days. Patients may be asymptomatic or may have vomiting, diarrhea, and dehydration. Diarrhea is usually self-limited and lasts 4–7 days. Symptoms of URI may sometimes be present.
3. Diagnosis. A positive stool enzyme-linked immunosorbent assay (ELISA) test is used to make the diagnosis. WBCs and blood are typically absent from the stool.
4. Management. Treatment is supportive with particular attention to fluid management and early institution of feedings to prevent gut atrophy. Some children may develop transient lactose intolerance.
b. Norovirus
1. Epidemiology. Norovirus is an RNA virus, which also is very efficiently spread by the fecal–oral route and through contact with infected fomites. Outbreaks of gastroenteritis occur in all age groups, particularly in closed populations (e.g., day care centers, schools, cruise ships).
2. Clinical features are similar to those caused by rotavirus with vomiting as a prominent symptom. Duration of illness is only 48–72 hours, a shorter duration as compared with the other viral causes.
3. Diagnosis is confirmed by enzyme immunoassays for stool antigen detection, as well as norovirus-specific PCR.
4. Management is supportive with close attention to hydration status and electrolytes.
2. Bacterial causes, associated clinical features, and management (see Table 7-6)
3. Parasitic causes [see section XV] B. Evaluation should include a detailed history, complete physical examination, and selective laboratory studies.
1. Specific historical features include the following:
a. Presence of fever, rash, abdominal pain, vomiting, and blood or mucus in the stool
b. Recent antibiotic use (e.g., may result in C. difficile infection)
c. Day care attendance or travel
d. Pets (e.g., lizards and turtles as well as domesticated fowl may transmit Salmonella
species)
e. Unusual foods or recently consumed recalled foods (e.g., raw dairy product consumption has been associated with outbreaks of Salmonella and E. coli, and well- publicized outbreaks of gastroenteritis have been linked to some packaged fruits, vegetables, nuts, and prepared foods).
2. Physical examination should be detailed and focused, especially on assessment of

hydration, particularly in young children.
3. Laboratory studies may include CBC, serum electrolytes, and assessment of stool for gross or occult blood (i.e., stool guaiac or immunoassay), WBCs (i.e., Wright stain), ova and parasite evaluation (three separate ova and parasite stool specimens increase the yield), and culture. Specific and rapid testing (ELISA or PCR) are useful in the detection of rotavirus, norovirus, Giardia lamblia, Cryptosporidium, and C. difficile infection. Bacterial multiplex PCR testing is now widely available to screen for bacterial diarrheal pathogens (Table 7-6).
a. The classic electrolyte finding is a non–anion gap hyperchloremic metabolic acidosis as a result of bicarbonate loss in the stool.
b. In general, the presence of either gross or occult blood in stools predicts the presence of stool WBCs.
c. The utility of a stool culture when WBCs are absent in the stool is low.
C. Management principles
1. Fluid management is the cornerstone of therapy.
2. Antibiotics are indicated for only a few causes of infectious diarrhea (Table 7-6) and should be avoided in cases of bloody diarrhea when enterohemorrhagic E. coli is suspected as antibiotics may increase the risk of hemolytic uremic syndrome.

Table 7-6
Characteristics of Bacterial Causes of Infectious Diarrhea

Bacterium Clinical Features Diagnosis Management
Enterotoxigenic           Major cause of traveler’s        Stool WBCs and RBCs

Escherichia coli           diarrheaGenerally                    absentDiagnosis is made noninvasive with watery                                                   clinically but can be diarrhea                         confirmed on culture

Antibiotics (fluoroquinolones or azithromycin in children) may shorten duration of

symptoms

Hydration is essential
Enteropathogenic        Noninvasive watery                Stool WBCs

E. coli (EPEC)              diarrhea seen in infants           absentDiagnosis made on and preschoolers       stool culture

Oral sulfonamides or fluoroquinolones are

indicated

Hydration is essential
Enterohemorrhagic Strain 0157:H7 is

E. coli (EHEC)             responsible for hemolytic

uremic syndrome (HUS)

via endotoxin release

Stool WBCs present If suspected or confirmed, antibiotic therapy is avoided (antibiotics may increase risk for or worsen the course of HUS as a result of enhanced endotoxin release)
Diagnosis confirmed by

culture and multiplex PCR

HUS: renal failure, thrombocytopenia, and microangiopathic

hemolytic anemia

Shigella sonnei            Bloody diarrhea

predominatesChildren may develop seizures secondary to neurotoxin release

Stool WBCs present Third-generation cephalosporins, macrolide (azithromycin), or fluoroquinolones are indicated
Culture is diagnostic and important for organism

susceptibilities

Multiplex PCRs available for multiple organism

detection

Salmonella species May cause bloody or

nonbloody diarrhea

Stool WBCs may be

present or absent

Treatment is not indicated for uncomplicated gastroenteritis in immunocompetent hosts

>3 months of age because it may increase carriage time

Spread by fecal–oral route, poultry, milk, eggs, and exposure to reptiles Culture is usually diagnostic and important for organism

susceptibilities

Patients, especially those with sickle cell disease, may develop bacteremia or

osteomyelitis

Multiplex PCRs available for multiple organism detection Treatment for invasive disease or in high risk host includes a third- generation cephalosporin
Campylobacter jejuni Most common cause of

bacterial bloody diarrhea in the United States

Stool WBCs are usually           Oral macrolide (azithromycin and

present if blood is                                                  erythromycin) is indicated and leads to presentStool culture is                                   improved symptoms and decreased risk for diagnostic as is multiplex        relapse, but symptoms commonly resolve

Disease is often self-
  limited and is spread by contaminated food

(usually poultry)

PCR without antimicrobial intervention
Yersinia enterocolitica May cause mesenteric adenitis along with gastroenteritis that may

mimic acute appendicitis

Stool culture or mesenteric node culture grows the organism Antibiotics may benefit patients; third- generation cephalosporins are commonly used
Clostridium difficile A normal component of gut flora that may cause colitis if it overgrows the rest of the gut flora, as is seen after antibiotic use Diagnosis is made by identifying toxin in the

stool

Oral or intravenous metronidazole is effective
Endoscopy may demonstrate

pseudomembranes

Oral vancomycin is reserved for resistant cases
Vibrio cholerae Seen in developing countriesCharacterized by watery diarrhea with massive water loss Diagnosis is based on history of massive watery diarrhea in a patient returning from, or residing in, an endemic

area

Fluid replacement is criticalAntibiotics (macrolides, fluoroquinolones, and tetracyclines) are recommended in conjunction with fluid replacement to shorten the duration of diarrhea and reduce viral shedding.
V. cholerae may be cultured from stool, but this is not routinely performed in the

United States

Detection of cholera toxin gene by EIA and serologic testing is available at the Centers for Disease

Control and Prevention.

WBCs = white blood cells; EIA = enzyme-linked immunoassay; PCR = polymerase chain reaction; RBCs = red blood cells.

XII. Urinary Tract Infections (see Chapter 11, section XIV)

XIII. Specific Viral Infections
A. HIV (Human Immunodeficiency Virus)
1. Epidemiology. The number of children less than 13 years of age infected with HIV significantly declined with the introduction of interventions to prevent mother-to-infant transmission. Half of all new HIV infections in the United States occur in people less than 25 years of age; thousands of teens acquire new HIV infections each year. Worldwide, over 3 million children are living with HIV, the majority of whom were infected perinatally or through breastfeeding. The highest incidence of pediatric HIV is in sub- Saharan Africa.
2. Transmission
a. Perinatal transmission in the United States and other developed countries has decreased dramatically due to interventions that prevent mother-to-child transmission.
1. In utero, intrapartum, or postpartum (through breastfeeding) transmission of HIV from an infected mother to her infant may occur. Transmission rates range from <2% in the United States, because of the common use of antiretroviral therapy during pregnancy, to 50% in the developing world where access to antiretroviral therapy may be limited.
2. Factors that increase the risk of transmission:
a. High maternal viral load (as measured by number of RNA copies)
b. Advanced maternal HIV disease
c. Primary maternal HIV infection
d. Concomitant maternal genital infections, including chorioamnionitis
e. Premature birth
f. Prolonged rupture of membranes
3. Factors that decrease the risk of transmission:
a. Undetectable maternal viral load
b. Cesarean section
c. Adherence to maternal highly active antiretroviral therapy (HAART) and infant postexposure prophylaxis with antiretroviral therapy (zidovudine)
b. Other modes of HIV transmission are as follows:
1. Sexual contact, an important mode of infection in adolescents and young adults
2. Blood product transmission, which is now exceptionally rare owing to very sensitive mandatory blood product screening
3. Sharing intravenous and tattoo needles
3. Clinical features
a. Most infants with perinatally-acquired HIV infection are asymptomatic for the first year of life.
b. The following are the early symptoms of HIV infection in children:
1. Failure to thrive
2. Thrombocytopenia
3. Recurrent infections, such as otitis media, pneumonia, and sinusitis
4. Lymphadenopathy
5. Parotitis
6. Recurrent, difficult-to-treat thrush
7. Loss of developmental milestones
8. Severe varicella infection or zoster

4. Diagnosis
a. All infants born to HIV-infected mothers will have transplacentally acquired maternal antibody that may persist for as long as 18–24 months.
b. HIV-specific DNA PCR is performed at birth, 2 weeks, and between 2 and 4 months of age to detect infants who are infected perinatally.
c. Negative HIV-specific DNA PCR at 4 months is consistent with an infant who has not been infected. If the DNA PCR is negative for HIV, infants are followed for the next year and tested for HIV by EIA after they have lost transplacentally acquired maternal antibody (by age 18–24 months).
5. Management
a. Infants born to HIV-infected mothers should be tested for the presence of virus, as outlined earlier in section XIII.A.4, in addition to the following:
1. Zidovudine administered orally for 6 weeks for postexposure prophylaxis.
2. Trimethoprim/sulfamethoxazole (TMP/SMX) for Pneumocystis jirovecii pneumonia (PCP) prophylaxis is started at 6 weeks of age until HIV DNA PCR at age 4 months is negative.
3. No breastfeeding if safe water supply is available (developed countries)
4. Urine CMV culture to detect coinfection with CMV (occurs in 5%)
5. CBC with differential to monitor for zidovudine toxicity (bone marrow suppression)
b. HIV-infected children should be managed at an institution experienced in the care of children with HIV. Management includes administration of medications, nutritional counseling, social work services, and regular neurodevelopmental testing.
1. All HIV-infected children should receive antiretroviral agents that may include nucleoside reverse transcriptase inhibitors (NRTIs), non–nucleoside reverse transcriptase inhibitors (NNRTIs), and protease inhibitors, as well as newer agents that include integrase strand transfer inhibitors and CCR5 (chemokine receptor 5) antagonists. Combination therapy is the cornerstone of treatment to avoid selection of resistant viruses. Close monitoring is essential because medications may cause bone marrow suppression, hepatitis, and pancreatitis.
2. Prophylaxis for opportunistic infections is important. The decision to begin prophylaxis is based on the patient’s age, CD4 count, and history of prior opportunistic infections.
3. Immunizations and usual well-child care are critical. HIV-infected children should receive all routine childhood vaccines except the live varicella vaccine. The measles, mumps, and rubella (MMR) vaccine, although a live viral vaccine, is currently recommended for all but the most severely immunocompromised HIV-infected children. Annual influenza vaccine, pneumococcal vaccine (both conjugate, PCV-13, and polysaccharide, PPSV- 23), as well as annual testing for tuberculosis are all recommended. All family members should receive influenza immunization to protect immunocompromised household members.
4. Regular monitoring of T-cell subsets and HIV RNA PCR to assess viral load are used to monitor response to therapy, typically every 3 months.
5. Ophthalmologic examination to assess for CMV retinitis in HIV-infected children who are CMV antibody positive should occur annually.
6. Complications of HIV infection
a. Opportunistic infections

1. PCP
a. Epidemiology
1. Most common opportunistic infection in HIV-infected children
2. Risk of infection correlates with CD4 cell number and percentage: for children <5 years of age, a CD4 count <500 cells/µL or percentage <15% and for children ≥5 years of age, a CD4 count
<200 cells/µL or percentage <15%.
b. Clinical features. Fever, hypoxia, and interstitial pulmonary infiltrates are present.
c. Management. Prophylaxis against PCP infection is with oral TMP/SMX and is based on the patient’s age and CD4 cell count and percentage. Treatment of PCP infection may include TMP/SMX, dapsone, pentamidine, or atovaquone. Moderate to severe infections are also treated with systemic corticosteroids.
2. M. avium complex (MAC). MAC is characterized by fever, weight loss, night sweats, abdominal pain, bone marrow suppression, and elevated transaminases. Risk is highest when the CD4 count falls less than 50 cells/mm3.
3. Fungal infections. Candidal infections (e.g., thrush, esophagitis), cryptococcal infections (e.g., meningitis, pneumonia), histoplasmosis, coccidioidomycosis, and aspergillosis may occur.
4. Viral infections. CMV (e.g., retinitis, esophagitis, colitis), HSV, and VZV infections may occur.
5. Parasitic infections. Toxoplasmosis and infections caused by Cryptosporidium
and Isospora belli may occur.
b. Lymphoma, caused by primary or reactivation of EBV infection
7. Prognosis. The expanded testing of pregnant women for HIV, coupled with aggressive prenatal antiretroviral therapy, has dramatically reduced and almost eliminated perinatal HIV transmission in the United States. Morbidity and mortality of HIV-infected children have also declined, correlating with the licensure of highly active antiretroviral agents and more sensitive testing for better evaluation of response to therapy.
B. Infectious mononucleosis
1. Etiology and epidemiology
a. EBV, a member of the herpes virus family, is the major etiologic agent. EBV is commonly acquired during adolescence, although infection also occurs often in young children. EBV is transmitted primarily by saliva and infects the B lymphocytes.
b. Other agents, including toxoplasmosis, CMV, and HIV, may cause a similar clinical syndrome.
2. Clinical features
a. Young children may be asymptomatic.
b. Older children develop typical signs and symptoms.
1. Fever, which may last up to 3 weeks
2. Malaise and fatigue
3. Tonsillopharyngitis (typically exudative, resembling GABHS pharyngitis)
4. Cervical lymphadenopathy (lymphadenopathy may also be diffuse)
5. Hepatosplenomegaly. The spleen is enlarged in 80% of patients.
6. A minority of patients may also develop a macular rash.
7. Symptoms self-resolve in weeks to months.
3. Diagnosis
a. CBC may demonstrate atypical lymphocytes. Other laboratory abnormalities

include neutropenia, thrombocytopenia, and elevated transaminases.
b. Monospot is a first-line test in diagnosing EBV infectious mononucleosis. A monospot measures the presence of heterophile antibody, which has the ability to agglutinate sheep RBCs. It has an overall sensitivity of 85% but is less sensitive in children under 4 years of age because these antibodies do not reliably form in younger children. CMV causes most monospot-negative cases of infectious mononucleosis in older children.
c. EBV antibody titers are the preferred method of diagnosing EBV infection in children younger than 4 years of age.
1. To diagnose EBV, antibodies to viral capsid antigen (VCA), early antigen (EA), and Epstein–Barr nuclear antigen (EBNA) are tested.
2. Acute infection is diagnosed by finding elevated levels of IgM–VCA and absent antibodies to EBNA. Antibodies to EBNA are detected 2–3 months after acute infection. If EBNA is positive at the time of evaluation, the symptoms are not due to EBV (a way to remember this is EBNA = Epstein–Barr Not Active).
d. PCR testing is the test of choice for evaluating immunocompromised patients suspected of having complications of EBV primary disease or reactivation, but should not be used in healthy children to make an acute EBV diagnosis.
4. Management. Therapy for most cases of EBV infection is supportive. Corticosteroids are sometimes used for severe tonsillopharyngitis causing airway compromise. Antivirals are not used to treat EBV in immunocompetent patients, as they are not active against the latent form of the virus.
5. Complications
a. Neurologic complications, including cranial nerve palsies and encephalitis
b. Severe tonsillopharyngitis, which may cause upper airway obstruction
c. Ampicillin-associated rash. Patients with EBV infection who are misdiagnosed with GABHS pharyngitis and prescribed ampicillin (or amoxicillin or other beta- lactam antibiotics) often develop a diffuse pruritic maculopapular rash one week after starting the antibiotic. This is not an allergic reaction but is idiosyncratic.
d. Splenic rupture. Children with infectious mononucleosis with splenomegaly should be restricted from contact sports until the spleen has returned to normal size, typically after 3–4 weeks.
e. Hemophagocytic lymphocytosis or hemophagocytosis, a condition in which bone marrow cells are consumed by activate macrophages, can result following primary infection with EBV. Clinical features of this serious condition include fever with multiple organ involvement. Hallmark laboratory findings include extremely elevated ferritin, transaminitis, and coagulopathy.
f. Malignancy. EBV has been isolated from nasopharyngeal carcinoma and Burkitt lymphoma. Patients who have received organ transplants are at risk for EBV- associated posttransplant lymphoproliferative diseases.
C. Measles
1. Etiology. Measles is also known as rubeola or “10-day measles” (in contrast to rubella or “3-day measles”) and is caused by an RNA virus of the Paramyxoviridae family.
2. Epidemiology
a. Routine measles vaccination has significantly decreased and almost eliminated the incidence of measles in the United States over the past 50 years. Before the vaccination program, there were millions of cases of measles annually. Widespread immunization has led to eradication in the United States. Approximately 50–100 cases per year in the United States are due to importations from measles endemic

countries leading to outbreaks among unimmunized individuals.
b. Measles is highly infectious and spreads easily among susceptible individuals in households and schools.
3. Clinical features. Manifestations develop after an 8- to 12-day incubation period. Clinical features include a classic clinical prodrome, followed by a transient enanthem (rash on mucous membranes) and then a characteristic exanthem (rash on the skin).
a. The three Cs (cough, conjunctivitis, and coryza) is a mnemonic to help remember the classic prodrome. Other early symptoms include photophobia and low-grade fever.
b. The enanthem (Koplik spots) is characterized as small gray papules on an erythematous base, located on the buccal mucosa. Koplik spots are pathognomonic of measles and are present before the generalized exanthem. They are transient and may be absent by the time the other typical clinical features develop.
c. The exanthem is characterized as an erythematous macular eruption that begins around the neck and ears and spreads down the chest and upper extremities during the subsequent 24 hours. The exanthem covers the lower extremities by the second day, becomes confluent by the third day, and lasts for 4–7 days.
d. Temperature usually >101°F (>38.3°C) accompanies the onset of symptoms.
4. Complications
a. Pneumonia is the most common complication and the most common cause of mortality.
b. Otitis media is also common.
c. Laryngitis and laryngotracheobronchitis (croup)
d. Encephalomyelitis (i.e., inflammation of both the brain and spinal cord)
e. Subacute sclerosing panencephalitis is a rare late complication of measles infection that occurs years after the primary infection. Children infected in the first two years of life are at greatest risk for developing this debilitating and often fatal disease.
5. Diagnosis. The basis of diagnosis is clinical features and confirmation of measles infection by PCR testing of samples from the urine, throat, or nasopharynx, or by detection of measles-specific IgM.
6. Management
a. Supportive care is most important.
b. Vitamin A has been shown to improve outcomes of children in developing countries, and it is recommended that all hospitalized infants and children with measles be treated with vitamin A.
c. Immunoglobulin can be used for postexposure prophylaxis in high-risk individuals (e.g., children with HIV and other immunodeficiency states, or infants under
12 months of age) who are exposed to measles.
D. Rubella
1. Etiology. Rubella, which is also known as German measles or “3-day measles,” is caused by an RNA virus in the togavirus family.
2. Epidemiology. Like measles, the incidence of rubella has declined during the past 50 years as a result of routine immunization during childhood and is no longer circulating in the United States. Rubella is also highly infectious.
3. Clinical features. Unlike measles, rubella is a mild disease and is often asymptomatic. Incubation period is 14–21 days.
a. The prodrome includes mild upper respiratory symptoms and low-grade fever.
b. Painful lymphadenopathy, especially of the suboccipital, posterior auricular, and cervical nodes

c. The exanthem follows the adenopathy and is characteristically nonpruritic, maculopapular, and confluent. It begins on the face, spreads to the trunk and extremities, and lasts 3–4 days.
d. The fever is usually mild (<101°F or <38.3°C) and accompanies the other clinical symptoms.
4. Complications
a. Meningoencephalitis
b. Polyarthropathy, involving mostly the fingers, knees and wrists, is seen primarily in teenage girls and young women, and may last several weeks.
c. Congenital rubella syndrome (CRS) is the most serious complication of an otherwise relatively benign disease. Owing to vaccination, CRS has been successfully eliminated from the Americas.
1. CRS occurs after primary maternal infection during the first trimester. Fetal anomalies occur in 30–50% of infected fetuses.
2. Presenting clinical features include thrombocytopenia, hepatosplenomegaly, jaundice, and purpura (“blueberry muffin baby”).
3. Structural abnormalities include congenital cataracts and a patent ductus arteriosus. Other findings include sensorineural hearing loss and meningoencephalitis.
4. Late complications may include mental retardation, hypertension, type 1 diabetes mellitus, and autoimmune thyroid disease.
5. Diagnosis is by viral culture or PCR or by serology. Because of transplacental passage of IgG antibody, diagnosis of an infant with CRS is made by viral isolation or by PCR.
6. Management is supportive.
E. Hepatitis (see Chapter 10, section XI.F)
F. Varicella (see Chapter 19, section V.C.6)

XIV. Specific Fungal Infections
A. Aspergillosis. Aspergillus species are ubiquitous molds that cause both invasive disease and noninvasive allergic disease.
1. Invasive disease occurs in severely immunocompromised patients, such as recipients of bone marrow or solid organ transplant. Management includes high-dose systemic antifungal therapy with amphotericin B and often surgery to resect the aspergilloma, a tumor-like mass formed by the fungus. Prognosis is poor.
2. Allergic bronchopulmonary aspergillosis is characterized by wheezing, eosinophilia, and pulmonary infiltrates. It occurs most commonly in patients with chronic lung disease (e.g., cystic fibrosis). Patients have elevated aspergillus-specific immunoglobulin E levels, and management includes corticosteroids and, in some cases, antifungal therapy.
B. Candidiasis
1. Epidemiology and etiology. Candida species, especially C. albicans, are present on the skin and throughout the gastrointestinal tract.
a. In immunocompetent individuals, overgrowth of yeast may occur normally, or under the influence of systemic antibiotics, causing mild superficial infection.
b. In immunocompromised individuals, overgrowth of yeast may occur readily, causing severe invasive infection.
2. Clinical features and management
a. Overgrowth of Candida on the skin or mucous membranes may lead to diaper dermatitis, oral thrush, or vulvovaginal candidiasis. Treatment includes topical antifungal therapy.
b. Invasive candidal infections in immunocompromised patients may include fungemia, meningitis (see Table 7-3 for CSF profile), osteomyelitis, and endophthalmitis. Treatment includes systemic antifungal therapy.
C. Coccidioidomycosis
1. Etiology. C. immitis is a fungus found in the soil in the southwestern United States and in Mexico.
2. Clinical features
a. Infection occurs when Coccidioides is inhaled into the lungs.
b. Most infections are asymptomatic or cause a mild pneumonia.
c. African Americans, Filipinos, pregnant women, neonates, and immunocompromised individuals are at highest risk for disseminated disease, which may include severe pneumonia, meningitis, and osteomyelitis.
3. Management. Mild pulmonary disease in immunocompetent patients generally does not require treatment. Disseminated disease and illness in immunocompromised hosts are treated with systemic antifungal therapy.
D. Cryptococcal infection
1. Etiology. Cryptococcus neoformans is a yeast found in the soil.
2. Clinical features
a. Infection is acquired when Cryptococcus is inhaled into the lungs.
b. Most infections are asymptomatic.
c. Spread to the central nervous system (CNS) occurs primarily in immunocompromised patients. Cryptococcal meningitis is one of the AIDS- defining illnesses.
d. Disseminated infection, including infection of the bones, joints, and skin, may also occur in immunocompromised hosts but is rare in children.

3. Management. Treatment of disseminated and CNS cryptococcal infection includes systemic antifungal therapy.

XV. Specific Parasitic Infections
A. Amebiasis
1. Etiology. Infection is by the protozoan Entamoeba histolytica. Infection is acquired by ingestion of the cyst in contaminated food or water. Symptoms begin 1–4 weeks later as the trophozoite form emerges from the cyst and invades the colonic mucosa.
2. Epidemiology. Amebiasis is present worldwide with the highest incidence in developing nations.
3. Clinical features
a. Most patients are asymptomatic.
b. Symptomatic intestinal disease ranges from mild colitis to severe dysentery. Young children, pregnant women, and immunocompromised patients have more severe disease.
1. Symptoms include cramping abdominal pain, tenesmus, and diarrhea that may contain blood or mucus. Weight loss, fever, tender hepatomegaly, chest pain, right shoulder pain, respiratory distress, and jaundice may also occur.
2. Abdominal complications include intestinal perforation, hemorrhage, strictures, and a local inflammatory mass or ameboma.
c. Extraintestinal amebiasis manifests as an abscess, most commonly in the liver, although it may form in the brain, lung, or other organs.
4. Diagnosis. Identification of the trophozoites or cysts in the stool is diagnostic. Colonoscopy with biopsy or serum antibody assays may also be helpful. Ultrasound or CT scan can identify an abscess in the liver or other organs.
5. Management. Treatment is based on the site of involvement and includes elimination of both the invading organism and those within the intestinal lumen. Metronidazole is the mainstay of therapy and is recommended along with a luminal amebicide, such as iodoquinol.
B. Giardiasis
1. Etiology. Infection is by the protozoan G. intestinalis. Infection occurs by fecal–oral contamination when the cyst is accidentally ingested.
2. Epidemiology. Giardiasis occurs worldwide, and it is the most common intestinal parasitic infection of humans. Individuals who drink contaminated mountain water in the western United States are at higher risk, but giardiasis is an endemic disease and can occur as large waterborne or day care center outbreaks. Humans are the primary reservoirs, and transmission is commonly person-to-person, but can also be from animals, such as dogs and cats.
3. Clinical features. Signs and symptoms are variable and range from asymptomatic disease to explosive diarrhea. Symptoms occur 1–2 weeks after ingestion of the cyst and may persist for 2–6 weeks.
a. Infection localizes within the small bowel, causing diarrhea that is typically described as voluminous, watery, and foul smelling.
b. Abdominal pain, cramping, bloating, flatulence, weight loss, and low-grade fever may also occur.
4. Diagnosis. Direct examination of stool for cysts and trophozoites or stool ELISA tests are used to make the diagnosis. Small bowel biopsy is sometimes indicated in difficult- to-diagnose cases.
5. Management. Treatment includes metronidazole, tinidazole, or nitazoxanide.
C. Malaria
1. Etiology. Malaria is an obligate intracellular blood-borne parasitic infection caused by

four species of Plasmodium: Plasmodium falciparum (responsible for the most severe disease), Plasmodium vivax, Plasmodium malariae, and Plasmodium ovale.
2. Epidemiology
a. Malaria is the most important parasitic cause of morbidity and mortality in the world, responsible for 2–3 million deaths each year, mostly in young children.
b. Malaria is endemic in tropical and subtropical regions of the world. The risk of malaria is high for travelers to these endemic areas.
c. Transmission of Plasmodium occurs via the bite of the infected female Anopheles
mosquito, a night-biting mosquito.
3. Clinical features
a. Initial findings include vague flulike symptoms that typically include headache, malaise, anorexia, and fever.
b. Cyclical fevers follow the flulike prodrome and occur every 48–72 hours, which correlate with RBC rupture and subsequent parasitemia. Chills, vomiting, headache, and abdominal pain may also occur.
c. Other features include hemolytic anemia, splenomegaly, jaundice, and hypoglycemia. Cerebral malaria, renal failure, shock, and respiratory failure may all occur.
4. Diagnosis. Identification of the parasite on thin and thick Giemsa-stained peripheral blood smears is diagnostic. The thick smear is for malarial screening, and the thin smear is for malarial identification and staging (determination of the level of parasitemia) of the particular Plasmodium species. Rapid diagnostic testing is becoming increasingly more available, and is particularly important in resource poor areas. Molecular (PCR) tests are also available in reference laboratories. The U.S. Centers for Disease Control and Prevention has consultants available who can assist with diagnosis of patients with suspected malaria.
5. Management. Choice of antimalarial therapy is based on resistance patterns, species type, and severity of illness. Medications include chloroquine, atovaquone-proguanil, quinine, quinidine gluconate, mefloquine, doxycycline, and clindamycin used in combination with quinine.
6. Prevention
a. Avoidance of mosquito bites is the mainstay of prevention. DEET (N,N-diethyl-m- toluamide)-containing repellants, insecticide-impregnated bed nets, and protective clothing are essential.
b. Chemoprophylaxis, which may include oral chloroquine, mefloquine, doxycycline, or atovaquone and proguanil hydrochloride (depending on the area visited and its Plasmodium species and resistance patterns).
c. Control of the Anopheles mosquito is important for individuals living in endemic areas.
D. Toxoplasmosis
1. Etiology. Infection is by the intracellular parasite T. gondii.
2. Epidemiology. Transmission occurs through direct contact with cat feces (the cat is the definitive host for the parasite); ingestion of undercooked meat, fruits, or vegetables contaminated with cysts; transplacental passage; exposure to contaminated blood products; or organ transplantation.
3. Clinical features
a. Most patients are asymptomatic.
b. Symptoms, if present, include a mononucleosis-like illness consisting of malaise, fever, sore throat, myalgias, and lymphadenopathy. Rash and hepatosplenomegaly may be present. Symptoms are self-limited and generally benign.

c. Reactivation of disease may occur if patients become immunosuppressed. In this case the presentation is often more severe and may include encephalitis, focal brain lesions, pneumonitis, or, rarely, disseminated disease. Toxoplasmosis is an important opportunistic infection in HIV-infected patients who commonly present with focal seizures.
d. Ocular toxoplasmosis may occur. T. gondii is the most common cause of infectious chorioretinitis.
e. Congenital toxoplasmosis is characterized by the triad of hydrocephalus, intracranial calcifications, and chorioretinitis.
4. Diagnosis. Serologic testing, PCR, or identification of the organism in cultures of amniotic fluid, CSF, or blood are used for diagnosis.
5. Management. Most infections do not require specific therapy. Treatment is indicated for infants with congenital toxoplasmosis, pregnant women with acute toxoplasmosis, and immunocompromised individuals with reactivation resulting in toxoplasma encephalitis. Treatment includes the use of sulfadiazine and pyrimethamine.
6. Prevention. Pregnant women and immunocompromised individuals are at highest risk. Therefore, they should avoid cat feces and undercooked meats and should clean all fruits and vegetables before consumption. Gloves should be used when gardening or preparing meat.

XVI. Specific Helminth Infections
A. General concepts (characteristics of specific infections are noted in Table 7-7)
1. Groups at highest risk include immigrants, travelers, and homeless individuals.
2. Common clinical features
a. Most infections are asymptomatic.
b. Abdominal symptoms include pain, anorexia, nausea, rectal prolapse, and obstruction.
3. Diagnosis is usually made by three separate stool examinations for ova and parasites. To detect pinworms, a cellulose tape test may be performed; the tape is placed sticky side down on the perianal region before sleep and is removed immediately on awakening, and then examined for eggs.
B. Cysticercosis
1. Epidemiology
a. Worldwide distribution with high incidence in Mexico and Central America
b. In endemic areas, 20–50% of cases of epilepsy are caused by cysticercosis.
2. Etiology. Infection occurs via the fecal–oral route when the eggs of Taenia solium, the pork tapeworm, are accidentally ingested.
3. Clinical features
a. No symptoms are present until the tapeworm encysts in the muscle, subcutaneous tissue, or brain.
b. Subcutaneous nodules may be palpated or seen as calcifications on radiography.
c. Neurocysticercosis
1. The fourth ventricle is the most common site of involvement, although the brain parenchyma, meninges, spine, or eyes may be affected.
2. Signs and symptoms include seizures (presenting symptom in 70% of cases),
hydrocephalus, and stroke.
4. Diagnosis
a. Ova and parasite stool evaluation detects the Taenia eggs in only 25% of cases.
b. Serology is available in some laboratories.
c. Head CT or MRI scans may show a solitary parenchymal cyst, or single or multiple calcifications. Calcified lesions represent areas of old, nonviable parasitic infection.
5. Management. Antiparasitic medications are reserved for individuals infected with the adult tapeworm. Those with neurocysticercosis with brain imaging that shows only calcified lesions require only anticonvulsant therapy.

Table 7-7
Characteristics of Specific Helminth Infections

Infection Epidemiology Clinical Features Management
Enterobius vermicularis (pinworm) Common helminthic infection

in the United States

Anal or, less commonly, vulvar                Single dose of pyrantel pruritusInsomnia, anorexia, enuresis,                                                        pamoate or albendazole; nighttime teeth-grinding                                                                      repeat in 2 weeksTreat all

close contacts

Fecal–oral transmission of eggs
Preschool and school-age

children

Ascaris                          Largest and most common lumbricoides                                       intestinal roundwormFecal–oral (roundworms)                                       transmission of eggs Löffler syndrome—transient pneumonitis as larvae migrate through the lungs causing fever,

cough, wheezing, and eosinophilia

Single dose of albendazole or ivermectin, or 3-day treatment with mebendazole

or nitazoxanide

Small bowel obstruction Screen all close contacts
Trichuris trichiura        Worldwide distributionOften             Most are asymptomaticAbdominal (whipworm)                                   seen in association with Ascaris                                                            pain, tenesmus, bloody diarrhea, and Ivermectin or mebendazole for 3 days
infection                                                rectal prolapse Screen all close contacts
Necator americanus and Ancylostoma duodenale (hookworm) Rural, tropical, and subtropical          Rash and pruritus at site of

areas where soil is contaminated        penetration

with human feces

Albendazole, mebendazole,

or pyrantel pamoate

Iron supplementation
Percutaneous infection through a bare foot; larvae migrate to the lungs and are coughed up and

then swallowed

Iron-deficiency anemia with fatigue, pallor, and failure to thrive Screen close contacts
Strongyloides stercoralis Tropics, subtropics, and southern and southwestern United States Transient pruritic papules at site of

penetration

Ivermectin is the drug of choice; albendazole is associated with lower cure rates
Pneumonitis
Life cycle same as hookworm Gastrointestinal symptoms
Eosinophilia
Cutaneous larva migrans Intradermal migration of dog or

cat hookworms

Migrating, pruritic, serpiginous,

erythematous tracks on the skin

Resolves without treatment

in most cases

Contact with feces-contaminated

soil

Self-limited, lasting weeks to months Ivermectin or albendazole

for severe disease

Toxocara canis or cati (toxocariasis or visceral larva migrans; VLM) Most common in children 1–

4 years of age who have pica

Generalized VLM: fever,                                                             Albendazole eosinophilia, leukocytosis, and

hepatomegaly; may have malaise,

anemia, cough, or myocarditis

Ingestion of eggs in contaminated

soil or dog fur

Larvae released from eggs and migrate through tissues Ocular larva migrans: retinal granulomas or endophthalmitis Steroids may also be used for severe cases of myocarditis or CNS or

ocular involvement

XVII. Miscellaneous Infections
A. Rickettsial infections
1. Lyme disease (see Chapter 16, section VII)
2. Rocky Mountain Spotted Fever (RMSF)
a. Etiology. RMSF is caused by Rickettsia rickettsii, a Gram-negative intracellular coccobacillus that is transmitted by the bite of a tick.
b. Epidemiology
1. RMSF is endemic across the United States, but occurs primarily in the
southeastern regions of the United States.
2. Incidence is highest in school-age children, and infection usually occurs in the spring and summer.
3. Fewer than 50% of patients recall a tick bite.
c. Clinical features. Symptoms and signs range from mild to life-threatening, and may include:
1. Fever
2. Petechial rash that begins on the extremities (ankles and feet) and moves in a caudal and centripetal direction (i.e., wrists and hands first, and then to trunk and head)
3. Myalgias
4. Hepatosplenomegaly and jaundice
5. CNS symptoms, such as headache, coma, and seizures
6. Hypotension
d. Laboratory findings. Thrombocytopenia, elevated transaminases, and hyponatremia may occur. CSF findings may show an aseptic meningitis picture (see Table 7-3).
e. Diagnosis. The diagnosis is made clinically but should be confirmed with serologic tests for Rickettsia.
f. Management. Treatment includes oral or intravenous doxycycline and supportive care. Antibiotics are usually started empirically on the basis of clinical presentation before the results of diagnostic testing, given the risk of significant morbidity and mortality in untreated infection. Although doxycycline is not used in children
<8 years of age, given the life-threatening complications of this infection, doxycycline is administered to all children when RMSF is suspected.
g. Prevention. Prevention methods include tick avoidance and prompt tick removal. Prophylactic antibiotics after tick bites are not indicated.
3. Ehrlichia and anaplasma
a. Etiology. Human monocytic ehrlichiosis (HME) is caused by multiple organisms (Ehrlichia chaffeensis is the most prevalent) and is transmitted by the bites of several tick species. Human granulocytic anaplasmosis (HGA), also transmitted by tick bite, is caused by Anaplasma phagocytophilum.
b. Epidemiology. Most cases occur in the spring and summer, and the epidemiology follows the location of the transmitting tick. HME is found in the Southern and Southeastern United States, and HGA is primarily seen in the Northeast and the Midwest.
c. Clinical features. Ehrlichiosis and anaplasmosis are often referred to as “spotless RMSF” because they have many of the same presenting findings as RMSF but usually no rash. Signs and symptoms include fever, headache, myalgias, and lymphadenopathy.

d. Laboratory findings. These are similar to those seen in RMSF and other rickettsial diseases [see section XVII.A.2.d].
e. Diagnosis. The diagnosis is confirmed by serology as well as specific PCR.
f. Management. Treatment includes the use of doxycycline and supportive care.
B. Cat scratch disease
1. Etiology. Cat scratch disease is caused by the Gram-negative bacteria B. henselae.
2. Clinical features
a. Regional lymphadenopathy (especially in the axillary, cervical, or inguinal region), typically distal to the site of a cat scratch, or more commonly a kitten scratch, is the most common presentation. Kittens with fleas are thought to be at highest risk for transmission of this species of bacteria.
b. The initial scratch results in a papule along the line of the scratch, followed by lymphadenopathy 1–2 weeks later.
c. The involved lymph node is commonly erythematous, warm, and tender. Suppuration occurs in approximately 10%.
d. Fever may occur in one-third of patients.
e. Less common findings include Parinaud oculoglandular syndrome (conjunctivitis and preauricular lymphadenitis associated with B. henselae infection), encephalitis, osteomyelitis, hepatitis, and pneumonia.
f. Patients with FUO with hepatosplenic lesions should be evaluated for cat scratch disease.
3. Diagnosis. Histopathologic examination of tissue will demonstrate granulomas with microabscesses. Bartonella can be cultured from tissue as well, but most cases are diagnosed with Bartonella-specific antibodies and/or PCR, which is much more sensitive than serology.
4. Management. Treatment usually consists of supportive care as the lymphadenitis self- resolves. Antibiotics are generally reserved for patients with systemic disease or immunodeficiency. Antibiotics used include oral azithromycin, TMP/SMX, and ciprofloxacin.
C. Tuberculosis (TB)
1. Etiology. The cause of TB is M. tuberculosis.
2. Categories
a. Exposed person is the term used to describe an individual who has been in recent contact with an individual with contagious pulmonary TB. Physical examination, tuberculin skin test or interferon gamma release assay (IGRA), and chest radiograph are all normal.
b. Latent tuberculosis infection (LTBI) is the term used to describe an asymptomatic individual with a positive tuberculin skin test or IGRA, normal physical examination, and a chest radiograph that either is negative or shows healed infection.
c. Tuberculosis disease is the term used to describe an individual with signs and symptoms of TB with or without positive findings on chest radiograph [see section XVII.C.5]. Disease may be pulmonary or extrapulmonary, or both.
3. Epidemiology
a. TB is most common among urban, low-income, and nonwhite racial and ethnic groups, although it may be seen in children of any socioeconomic status.
b. Those at highest risk include immigrants from highly endemic regions of the world, health care personnel, homeless individuals, residents of institutions or correctional facilities, and individuals with immunodeficiency conditions (e.g., HIV, chronic disease, immunosuppressive medications, including TNF alpha

receptor antagonists or blockers).
c. Transmission of TB occurs by inhalation of small airborne droplets from an individual with contagious pulmonary TB. Children younger than 10 years of age are generally not contagious because their cough is minimal and their pulmonary lesions are usually small.
4. Clinical features
a. In LTBI, most children with a positive tuberculin skin test or IGRA result are asymptomatic and do not progress to TB disease. Infants younger than 12 months of age and postpubertal adolescents are at the greatest risk of developing disease.
b. Symptoms of TB disease include fever, chills, weight loss, cough, and night sweats.
c. Extrapulmonary TB disease may include the following:
1. Cervical lymphadenitis (scrofula), the most common form of extrapulmonary TB disease in children
2. Meningitis (for CSF findings, see Table 7-3)
3. Abdominal involvement (ileitis)
4. Skin and joint involvement
5. Skeletal disease, which may involve the vertebrae (Pott disease)
6. Disseminated or miliary disease
7. Renal involvement
5. Radiographic features of TB disease
a. Hilar or mediastinal lymphadenopathy
b. Lobar involvement, pleural effusion, or cavitary disease, which typically affects the upper lung segments.
6. Diagnosis. Diagnosis and categorization of TB infection are based on an individual’s risk for infection, Mantoux skin test (Tuberculin skin test, or TST), chest radiographic findings, and culture.
a. Tuberculin skin test (TST) or Mantoux skin test contains five tuberculin units of purified protein derivative (PPD).
1. It is administered intradermally and read 48–72 hours later by health care personnel trained in interpretation.
2. The tuberculin skin test becomes positive 2–12 weeks after exposure to TB.
3. A positive tuberculin skin test is identified by measuring the area of induration (not erythema), and is interpreted on the basis of clinical and individual risk factors, which include the following:
a. ≥5 mm is considered positive in children who have had close contact with an individual with TB, who have clinical or chest radiographic findings consistent with TB, or who are immunocompromised, including those with HIV.
b. ≥10 mm is considered positive if children are younger than 4 years of age, have a chronic medical condition, or live in an area endemic for TB.
c. ≥15 mm is considered positive in children older than 4 years of age who do not have other risk factors.
b. IGRAs can be used in children 2 years and older, and is preferred to TST for children 2 years of age and older who have been vaccinated with bacillus Calmette– Guérin (BCG).
1. Two IGRAs are available: QuantiFERON Gold In-Tube test measures total interferon gamma produced and T-SPOT TB test measures the number of cells that produce interferon gamma. Both require one blood test and measure ex vivo interferon gamma production from T lymphocytes in response to

stimulation with antigens specific to M. tuberculosis complex, which includes
M. tuberculosis and M. bovis. These antigens are not found in BCG and so IGRA testing can be useful in the evaluation of children older than 2 years of age who have received a BCG and may have a cross reactive TST. IGRAs have comparable sensitivity to TST but increased specificity.
2. IGRA results can be positive, negative, or indeterminate.
a. Positive: Patient should be considered infected with M. tuberculosis
complex.
b. Negative
c. Indeterminate: Result can be indeterminate due to issues with collection, the specific assay, or the patient’s immune response.
c. Definitive diagnosis involves the following:
1. Positive culture for M. tuberculosis from early morning gastric aspirates obtained by a nasogastric tube (gastric aspirate samples are preferred for diagnosis in children because children are generally unable to cough up sputum for culture, but instead swallow the sputum into the stomach), pleural fluid, CSF, or other body fluids.
2. Positive staining of fluid for acid-fast bacilli (AFB)
3. Positive histology (caseating granulomas) from a biopsy specimen
7. Management. Treatment is based on the TB category.
a. Patients with LTBI are treated with isoniazid (INH) for 9 months. Older adolescents, pregnant adolescents, and adults are also given daily pyridoxine (vitamin B6) to prevent neurologic complications of INH therapy.
b. Patients with TB are treated on the basis of the location of TB and the susceptibility pattern of the organism. Treatment generally includes 2 months of INH, rifampin, pyrazinamide, and ethambutol, followed by 4 months of INH and rifampin.

Review Test
1. A 13-day-old male infant presents with a fever (temperature up to 100.6°F [38.1°C]), mild irritability, and diminished appetite. His parents report no change in the number of wet diapers. Which of the following statements regarding this patient’s management or prognosis is correct?
A. Careful observation at home is appropriate because of the relatively low fever and normal urine output.
B. The risk of bacteremia in this patient is approximately 25%.
C. Intramuscular ceftriaxone and close home monitoring are appropriate after evaluation with a complete blood count, blood culture, urinalysis, and urine culture.
D. Irrespective of the results of initial laboratory testing, management should include intravenous antibiotics and hospitalization.
E. Bacteria likely to cause fever in this patient include Streptococcus pneumoniae and
Haemophilus influenzae type b.
2. A 10-month-old female infant with up-to-date immunizations presents with a fever (temperature up to 103.5°F [39.7°C]) for the past 3 days. She was previously healthy. Her parents report no symptoms other than the fever. On examination, she is well hydrated and appears nontoxic, and no focus of infection is identified. Which of the following is the next appropriate management step?
A. Complete blood count (CBC) and blood culture, and empiric oral antibiotics to cover likely causes of bacteremia
B. Urine culture and urinalysis, and if suggestive of a urinary tract infection, empiric antibiotics to cover likely urinary pathogens
C. No laboratory studies are indicated because the patient appears nontoxic.
D. Hospitalization and empiric intravenous cefotaxime
E. CBC, blood culture, urinalysis, urine culture, lumbar puncture, and chest radiograph; intramuscular ceftriaxone should be given because of the high risk of bacteremia.
3. A 2-year-old girl presents with fever. On examination, she has exudative pharyngitis, enlarged posterior cervical lymph nodes, and splenomegaly. Which of the following statements regarding her evaluation and management is correct?
A. Amoxicillin should be prescribed after performing a throat culture for suspected “strep throat.”
B. Monospot testing is highly sensitive and is the best test to make a diagnosis in this case.
C. Human immunodeficiency virus is the most likely cause of this infection.
D. Amoxicillin may result in a pruritic rash in this patient.
E. The patient should be administered corticosteroids that will lead to rapid improvement and resumption of full activity.
4. A 6-year-old girl is sent home from summer camp with a temperature of 101.3°F (38.5°C), stiff neck, photophobia, and headache. Lumbar puncture in the emergency department reveals the following results: white blood count 380 cells/mm3, with 65% polymorphonuclear cells and 35% lymphocytes; normal protein and glucose; and negative Gram stain. Which of the following pathogens is the most likely cause of her meningitis?
A. Neisseria meningitidis
B. Streptococcus pneumoniae
C. Enterovirus
D. Borrelia burgdorferi
E. Mycobacterium tuberculosis
5. A previously healthy 18-month-old girl is admitted to the hospital with fever (temperature up

to 102.8°F [39.3°C]), vomiting, and lethargy. She was well until 2 days ago, when she was diagnosed with a viral upper respiratory infection. Lumbar puncture to evaluate the cerebrospinal fluid shows the following results: white blood cells 3050 cells/mm3, with 98% polymorphonuclear cells; very low glucose; and elevated protein. Gram stain shows Gram- positive diplococci. Initial management should include which of the following?
A. Vancomycin and third-generation cephalosporin
B. Third-generation cephalosporin alone
C. Ampicillin and third-generation cephalosporin
D. Third-generation cephalosporin and acyclovir
E. Third-generation cephalosporin and corticosteroids
6. A 25-year-old woman is pregnant with her first child. The woman has human immunodeficiency virus (HIV) infection that was diagnosed 2 years before this pregnancy. Which of the following has been shown to increase her risk of transmitting HIV to her infant?
A. Treatment with highly active antiretroviral therapy during pregnancy and before delivery
B. Exclusive bottle formula feeding
C. Prolonged rupture of membranes
D. Birth by cesarean section
E. Orally administered zidovudine given to the infant after birth
7. An 8-year-old girl presents with sore throat, fever, and a rough sandpaper-like rash over her trunk and extremities. A throat culture is positive for group A β-hemolytic streptococcus. Treatment of her infection with antibiotics will prevent which of the following complications?
A. Reactive arthritis
B. Rheumatic fever
C. Poststreptococcal glomerulonephritis
D. Guillain–Barré syndrome
8. A 1-year-old girl presents with weight loss and a 2-week history of large, bulky, nonbloody, foul-smelling stools. She has been attending day care and recently received amoxicillin for an ear infection. Which of the following is the most likely cause of her diarrhea?
A. Entamoeba histolytica
B. Enterotoxigenic Escherichia coli
C. Clostridium difficile
D. Giardia lamblia
E. Norwalk virus
9. A 19-year-old boy, a college sophomore, presents with high fever, headache, cough, conjunctivitis, and a diffuse macular rash over his trunk and face. He is unsure of his immunization status. You suspect measles infection. Which of the following is correct regarding this diagnosis?
A. Vitamin A may improve his outcome.
B. Koplik spots would likely be present on examination of his mouth.
C. Mortality is most commonly caused by measles encephalitis.
D. Diagnosis is based on culture and direct fluorescent antigen testing.
E. Corticosteroids will decrease symptoms and improve outcome.

The response options for statements 10–14 are the same. You will be required to select one answer for each statement in the set.

A. Malaria Plasmodium species
B. Toxoplasma gondii
C. Giardia lamblia
D. Entamoeba histolytica

E. Coccidioides immitis
F. Cryptococcus neoformans
G. Aspergillus fumigatus
H. Candida albicans

For each clinical description, select the most likely cause.

1. At birth, a term infant is noted to have hydrocephalus and intracranial calcifications on computed tomography of the head. Eye examination reveals bilateral chorioretinitis.
2. A 5-year-old boy is admitted with a fever of unknown origin. An abdominal computed tomographic scan reveals a large hepatic abscess.
3. A 12-year-old girl with cystic fibrosis has an exacerbation of her disease and presents with wheezing, pulmonary infiltrates, and eosinophilia.
4. A 16-year-old boy is admitted to the hospital for a workup of cyclical fevers after a trip to India. His illness began with flulike symptoms.
5. An 18-month-old girl and three of her day care classmates present with 2 weeks of watery diarrhea and some weight loss.
6. A 2-year-old boy has a positive tuberculin skin test that measures 12 mm. It was placed during a routine well-child care visit. He is well, without fever, chills, cough, weight loss, or night sweats. No known tuberculosis contacts are identified. Which of the following statements regarding this patient’s management is correct?
A. A chest radiograph should be ordered because the tuberculin test is positive.
B. He should be placed into respiratory isolation immediately because he is likely to spread tuberculosis to others.
C. Isoniazid is not indicated because this tuberculin skin test is negative.
D. Triple drug therapy for tuberculosis should be started immediately.
E. Gastric aspirates should be ordered.

The response options for statements 16–20 are the same. You will be required to select one answer for each statement in the set.

A. Salmonella species
B. Shigella sonnei
C. Yersinia enterocolitica
D. Clostridium difficile
E. Campylobacter jejuni
F. Vibrio cholerae
G. Enterotoxigenic Escherichia coli
H. E. coli 0157:H7

Match the clinical description with the likely causative organism.

1. While visiting Monterey, Mexico, a 16-year-old boy develops watery, nonbloody diarrhea, without fever.
2. A 3-year-old boy presents with an acute onset of high fevers, bloody diarrhea, and a generalized tonic-clonic seizure. The stool Wright stain reveals sheets of white blood cells.
3. An 8-year-old girl presents with a 1-week history of diarrhea and low-grade fever. The family reports that they have recently acquired a pet turtle.
4. A 10-year-old boy is admitted to the hospital and taken directly to the operating room for suspected acute appendicitis. Surgeons discover a normal appendix but enlarged mesenteric lymph nodes.
5. A group of travelers to Bangladesh suddenly develop massive, watery, nonbloody diarrhea

that results in severe dehydration and electrolyte imbalance.

The response options for statements 21–24 are the same. You will be required to select one answer for each statement in the set.

A. Buccal cellulitis
B. Impetigo
C. Necrotizing fasciitis
D. Erysipelas
E. Staphylococcal scalded skin syndrome
F. Toxic shock syndrome

Match the clinical description with the likely diagnosis.

1. A 9-month-old girl with mild facial eczema has fever and a facial skin rash. The skin lesion is weepy with a honey-colored crust.
2. An unvaccinated 4-month-old boy has a blue color to his cheeks and a positive blood culture for Haemophilus influenzae type b.
3. An infant boy has fever, an erythematous skin rash, and a positive Nikolsky sign.
4. A 7-year-old girl develops fever and a rapidly expanding tender skin rash with a well- demarcated border.

Answers and Explanations
1. The answer is D [II.C.5 and Table 7-1]. Fever in an infant younger than 28 days of age must be taken very seriously because the neonate’s immune system is immature. As a result, the current appropriate management for any neonate with fever (temperature >100.4°F [>38°C]) includes a complete workup for serious bacterial infection (SBI) that includes evaluation of the blood, urine, and cerebrospinal fluid for evidence of bacterial infection; administration of empiric intravenous antibiotics; and hospitalization. The risk of SBI in a nontoxic infant younger than 3 months of age is approximately 1–7%. Usual bacteria resulting in infection in this age group include group B streptococcus, Escherichia coli, and Listeria monocytogenes.
2. The answer is B [II.D.3 and Table 7-1]. Because of the patient’s elevated fever, evaluation for a urinary tract infection, including urine culture and urinalysis, is indicated. Because the infant is completely immunized, risk of bacteremia is low enough not to warrant blood tests. If she were incompletely immunized (had not received her 2-, 4-, and 6-month vaccinations), a complete blood count and blood culture would be indicated and intramuscular ceftriaxone may be given either empirically, or only if the white blood count is ≥15,000 cells/mm3. Hospitalization is generally not required unless the patient is toxic in appearance, is dehydrated, or has poor ability to return to the physician for follow-up. Neither evaluation of spinal fluid nor a chest radiograph is indicated in this nontoxic patient without respiratory signs or symptoms. Neither intravenous antibiotics nor hospitalization is indicated because the infant is nontoxic and well hydrated.
3. The answer is D [XIII.B]. This patient’s clinical presentation with fever, lymphadenopathy, pharyngitis, and splenomegaly is most consistent with infectious mononucleosis. If a child with infectious mononucleosis is given amoxicillin, a diffuse pruritic rash may develop. Monospot testing is highly sensitive in older children, but heterophile antibodies do not reliably form in children younger than 4 years of age. Antibody titers are therefore the preferred diagnostic test in such young children. The most common cause of infectious mononucleosis is Epstein–Barr virus. Although supportive care is most appropriate, corticosteroids may be indicated for treatment of infectious mononucleosis complicated by airway compromise, but are not routinely recommended. Splenomegaly is not consistent with the diagnosis of streptococcal pharyngitis.
4. The answer is C [IV.C.3 and Tables 7-3 and 7-4]. This cerebrospinal fluid (CSF) evaluation is most consistent with aseptic meningitis, specifically viral meningitis. Enteroviruses are the most common cause of viral meningitis and most often occur during the summer and fall. Early in viral meningitis, the white blood cell (WBC) count in the CSF may demonstrate a polymorphonuclear cell predominance that shifts to a lymphocyte predominance within 24– 48 hours. The normal protein and glucose and negative Gram stain are also consistent with viral meningitis. Meningitis caused by Neisseria meningitidis or Streptococcus pneumoniae would be reflected by a higher CSF WBC count, lower glucose, and higher protein. Although patients with Lyme meningitis, which is caused by Borrelia burgdorferi, may present with an aseptic CSF profile, the onset is not as acute as in this patient. Patients with Mycobacterium tuberculosis present with a low to very low glucose and elevated protein level in the CSF.
5. The answer is A [IV.B.5]. Empiric therapy of presumed bacterial meningitis should include a third-generation cephalosporin and the addition of vancomycin until sensitivities are available, because of the high level of pneumococcal antibiotic resistance in many communities. Ampicillin is not indicated because this child is out of the age range at which Listeria infection occurs. Acyclovir is not indicated because the cerebrospinal fluid profile is most consistent with bacterial meningitis, not viral meningitis. Acyclovir is used to treat neonates with presumed herpes simplex meningitis, or encephalitis or in older children with

encephalitis. Corticosteroids are effective in reducing the incidence of hearing loss in Haemophilus influenzae type b meningitis but have not been shown to be effective for other bacterial pathogens.
6. The answer is C [XIII.A.2]. Factors that increase the risk of HIV transmission from mother to infant include high maternal viral load (measured by RNA copy number) at delivery, concomitant chorioamnionitis or other genital tract infections, primary or advanced maternal HIV infection, premature birth, and prolonged rupture of membranes. Transmission may also occur through breast milk. Transmission is decreased through the use of maternal antiretroviral therapy, newborn prophylaxis with antiretroviral agents (e.g., zidovudine), birth by cesarean section, and low maternal viral load.
7. The answer is B [IX.A.5.f]. This patient’s clinical presentation of a sandpaper-like rash associated with pharyngitis and fever is consistent with scarlet fever, caused by erythrogenic toxin–producing strains of group A β–hemolytic streptococcus (GABHS). Although there are multiple complications of GABHS infection, including rheumatic fever, glomerulonephritis, and reactive arthritis, only rheumatic fever will be prevented by treatment with antibiotics.
8. The answer is D [Tables 7-6 and 7-7, XV.B.3]. Infection with the protozoan Giardia lamblia is associated with bulky, foul-smelling stools, weight loss, and day care attendance. Entamoeba histolytica and Clostridium difficile generally cause bloody diarrhea. Escherichia coli infection generally results in short-term watery diarrhea. Day care attendance is also associated with Norwalk virus; however, symptoms of Norwalk virus infection generally last only 48–
72 hours.
9. The answer is A [XIII.C.3, XIII.C.4, XIII.C.5]. This patient’s presentation is most consistent with measles infection. Management includes supportive care, and vitamin A therapy may also be beneficial. Koplik spots are transient, and by the time the rash is present, Koplik spots are no longer appreciated. Bacterial pneumonia is the most common complication of measles infection and is the most common cause of mortality. Diagnosis is based on confirmation by serologic testing in the presence of typical clinical features. Corticosteroids do not play a role in the therapy of measles.
10. The answers are B, D, G, A, and C, respectively [XV.D.3, XV.A.3, XIV.A.2, XV.C.3, and XV.B.3]. The triad of intracranial calcification, hydrocephalus, and chorioretinitis is consistent with congenital toxoplasmosis, which is caused by Toxoplasma gondii. Entamoeba histolytica may result in asymptomatic infection or colitis. The most common extraintestinal complication is a liver abscess. Aspergillus infection may result in invasive disease or in noninvasive allergic disease characterized by wheezing, eosinophilia, and pulmonary infiltrates. This can occur in patients with severe asthma or with cystic fibrosis. Malaria classically presents with a flulike illness followed by the development of high fevers that cycle in 48- to 72-hour paroxysms. Giardia lamblia typically presents with bulky, large-volume, watery stools that eventually lead to weight loss. Outbreaks can occur in day care settings where children are in diapers.
11. The answer is A [XVII.C]. A tuberculin skin test is considered positive depending on a patient’s specific risk factors for acquisition of tuberculosis. A tuberculin skin test ≥10 mm is considered positive if the patient is younger than 4 years of age or if the patient resides or has lived in an area endemic for tuberculosis. Therefore, given that the tuberculin skin test is positive in this patient, a chest radiograph to evaluate for pulmonary tuberculosis is indicated. Children younger than 10 years of age with tuberculosis are unlikely to be contagious because of minimal cough and pulmonary involvement. Medications for tuberculosis disease (e.g., rifampin, isoniazid, pyrazinamide, ethambutol regimen) are indicated if the patient has signs and symptoms of tuberculosis. Gastric aspirates are indicated only if the chest radiograph reveals pulmonary disease.
12. The answers are G, B, A, C, and F, respectively [Table 7-6]. Enterotoxigenic Escherichia coli is the major cause of traveler’s diarrhea and results in nonbloody watery stools. Bloody stools

may result from infection with Salmonella, Shigella, Yersinia, Campylobacter, enterohemorrhagic
E. coli, and Clostridium difficile. Shigella may be associated with seizures caused by the release of a neurotoxin. Salmonella may be acquired by ingestion of contaminated poultry or by exposure to turtles and lizards that carry the organism. Yersinia may result in mesenteric adenitis that causes pain mimicking acute appendicitis. Infection with Vibrio cholerae generally occurs in developing countries and causes massive fluid loss from the gut. E. coli 0157:H7 causes bloody diarrhea and is the causative agent in hemolytic uremic syndrome.
13. The answers are B, A, E, and D, respectively [IX.A]. Impetigo typically presents with honey- crusted lesions on the face; it is caused by infection with Staphylococcus aureus and group A β- hemolytic streptococcus (GABHS). Buccal cellulitis is characterized by a bluish color to the cheeks of a young child; this condition is typically caused by infection with Haemophilus influenzae type b, which is identified on blood culture. Staphylococcal scalded skin syndrome is manifested by Nikolsky sign or the extension of bullae with lateral pressure applied to the skin. Fever, tender skin, and widespread bullae are present. Erysipelas is characterized by tender, erythematous skin, but the border is well demarcated. Necrotizing fasciitis and toxic shock syndrome are complications of infection with GABHS. Necrotizing fasciitis is a severe skin infection that involves multiple layers of tissue. Toxic shock presents with fever, sunburn rash, and multiorgan failure. Both of these complications are a result of toxin produced by the GABHS.